You are on page 1of 175

PDF download

JAB SAARA INDIA


VEDANTU PE ONLINE PADHEGA

Scholarship Admission Test


 WIN an assured Scholarship upto 100%
 Take the Online Test from the comfort of your home Register NOW
 It’s Absolutely FREE Limited Seats!
 For Class 6-12 CBSE | ICSE | JEE | NEET

W W W.V E D A N T U . C O M
BEST RESULTS
FROM ONLINE CLASSES

548 268 1 OUT OF 3


27
STUDENTS
46
STUDENTS
RANKS RANKS STUDENTS WHO CLEARED SCORED SCORED
ACROSS IN TOP JEE MAINS SECURED A ABOVE ABOVE
RANK IN JEE ADVANCED
INDIA 10K 600 550

Mumbai se Panchkula se Gurugram se Patiala se Maharashtra se Surat se


Dhruv Rambhia Madhav Goel Tanmay Gangwar Shaurya Bhatia Anjaney Pandey Tanishi Bhatt
AIR 43 AIR 208 AIR 227 682/720 670/720 670/720

JEE Adv. NEET

157 Students 171 Students


Scored above 90% in 12th Scored above 90% in 10th

Jharkhand se Dubai se
Akshat Kumar Navya Ratnan
98.20% 98.00%

12TH Boards 10TH Boards

Rajahmundry se Dahod se
Reshma M Ummehani N
99.4% 97.92%

State Boards

Scholarship Admission Test


WIN an assured Scholarship upto 100%

Register NOW
 Take the Online Test from the comfort of your home
 It’s Absolutely FREE Limited Seats!
 For Class 6-12 CBSE | ICSE | JEE | NEET

W W W.V E D A N T U . C O M
NCERT Solutions for Class 12
Maths
Chapter 7- Integrals

Exercise 7.1

1. Find an anti-derivative (or integral) of the following functions by the


method of inspection. sin 2x
Ans: We use the method of inspection as follows:
d 1 d
 cos 2x   2sin 2x    cos 2x 
dx 2 dx
d  1 
 sin 2x    cos 2x 
dx  2 
1
Thus, the anti-derivative of sin 2x is  cos 2x .
2

2. Find an anti-derivative (or integral) of the following functions by the


method of inspection. cos 3x
Ans: We use the method of inspection as follows:
d 1 d
 sin 3x   3cos3x   sin 3x 
dx 3 dx
d 1 
 cos3x   sin 3x 
dx  3 
1
Thus, the anti - derivative of cos 3x is sin 3x .
3

3. Find an anti-derivative (or integral) of the following functions by the


method of inspection. e 2x
Ans: We use the method of inspection as follows:
 e   2e 2 x  e 
d 2x 1 d 2x
dx 2 dx
d 1 
 e2 x   e2 x 
dx  2 
1
Thus, the anti-derivative of e 2 x is e2 x .
2

Class XII Maths www.vedantu.com 1


4. Find an anti-derivative (or integral) of the following functions by the
method of inspection.  ax  b 
2

Ans: We use the method of inspection as follows:


d
 ax  b   3a  ax  b 
3 2

dx
1 d
  ax  b    ax  b 
2 3

3a dx
d  1 3
  ax  b     ax  b  
2

dx  3a 
1
Thus, the anti-derivative of  ax  b  is  ax  b  .
2 3

3a

5. Find an anti-derivative (or integral) of the following functions by the


method of inspection. sin 2x  4e 3x
Ans: We use the method of inspection as follows:
d  1 4 3x 
  cos 2x  e    sin 2x  4e 
3x

dx  2 3 
 1 4 
Thus, the anti-derivative of  sin 2x  4e3x  is   cos 2x  e3x  .
 2 3 

6.   4e3x  1 dx
Ans:
  4e  1 dx
3x

 4  e3x dx  1dx
 e3x 
 4 xC
 3 
4
 e3x  x  C
3

 1
x  1  2  dx
2
7.
 x 
Ans:
2 1 
  x 2  dx
x 1 

Class XII Maths www.vedantu.com 2


   x 2  1 dx
x3
 xC
3

8.   ax2  bx  c  dx
Ans:
  ax  bx  c dx
2

 a  x 2dx  b  xdx  c 1.dx


 x3   x2 
 a    b    cx  C
 3  2
ax 3 bx 2
   cx  C
3 2

9.   2x2  ex  dx
Ans:
  2x  e  dx
2 x

 2  x 2dx   e x dx
 x3  x
 2   e  C
 3
2
 x 3  ex  C
3
2
 1 
10.   x   dx
 x
Ans:
2
 1 
  x  x  dx
 
 1 
   x   2  dx
 x 
1
  xdx   dx  21.dx
x
2
x
  log x  2x  C
2

Class XII Maths www.vedantu.com 3


x 3  5x 2  4
11.  dx
x2
x 3  5x 2  4
Ans:  dx
x2
   x  5  4x 2  dx
  xdx  51.dx  4  x 2dx
x2 4
  5x   C
2 x

x 3  3x  4
12.  dx
x
x 3  3x  4
Ans:  dx
x
 52 1
 
1

   x  3x  4x 2  dx
2

 
 32   12 
3  
x 4 x 
7

      C
x2

7 3 1
2 2 2
7 3
2
 x 2  2x 2  8 x  C
7

x3  x2  x  1
13.  dx
x1
x3  x 2  x  1
Ans:  dx
x 1
We obtain, on dividing:
   x 2  1 dx
  x 2dx  1.dx
x3
 xC
3

14.  1  x  xdx
Ans:  1  x  xdx

Class XII Maths www.vedantu.com 4


 3

   x  x 2  dx
 
1 3

  x 2 dx   x 2 dx
2 3 2 5
 x2  x2  C
3 5

15.  x  3x 2  2x  3  dx
Ans:  x  3x 2  2x  3 dx
 52 3 1

 3  2x  2x  3x 2 
2

 
5 3 1

 3 x dx  2  x dx  3 x dx
2 2 2

6 72 4 52 3

 x  x  2x  C
2

7 5

  2x  3cosx  e  dx
x
16.
Ans:   2x  3cos x  e  dx x

 2 xdx  3 cos xdx   e dx x

2x 2
  3  sin x   e x  C
2
 x 2  3sin x  e x  C

  2x  3sin x  5 x dx 
2
17.

  2x  3sin x  5 x dx 
2
Ans:
1

 2  x 2dx  3 sin xdx  5 x 2 dx


 3
2x 3  x2 
  3   cos x   5    C
3 3
 
 2 
3
2 10
 x 3  3cos x  x 2  C
3 3

Class XII Maths www.vedantu.com 5


18.  secx  secx  tan x  dx
Ans:  sec x  sec x  tan x  dx
   sec 2 x  sec x tan x  dx
  sec 2 xdx   sec x tan xdx
 tan x  sec x  C

sec2 x
19.  dx
cosec2 x
sec2 x
Ans:  dx
cosec2 x
1
2
  cos x dx
1
sin 2 x
sin 2 x
 dx
cos 2 x
  tan 2 xdx
  sec 2 xdx  1dx
 tan x  x  C

2  3sin x
20.  cos 2 x
dx
2  3sin x
Ans:  dx
cos 2 x
 2 3sin x 
    dx
 cos x cos x 
2 2

  2sec 2 xdx  3 tan x sec xdx


 2 tan x  3sec x  C

 1 
21. The anti – derivative of  x   equals
 x
1 13 1

A. x  2x 2  C
3

Class XII Maths www.vedantu.com 6


2 23 1 2
B. x  x  C
3 2
2 32 1

C. x  2x 2  C
3
3 32 1 12
D. x  x  C
2 2
Ans:
 1 
 x  
 x
3 1
1 1
 x2
x 2
  x dx   x dx 
2 2
 C
3 1
2 2
2 32 1

 x  2x 2  C
3
Thus, the correct answer is C.

d 3
22. If f  x   4x 3  4 such that f  2   0 then f  x  is
dx x
1 129
A. x4  3 
x 8
1 129
B. x 3  4 
x 8
1 129
C. x4  3 
x 8
1 129
D. x 3  4 
x 8
d 3
Ans: Given, f  x   4x 3  4
dx x
3
Anti-derivative of 4x 3  4  f  x 
x
3
 f  x    4x 3  4  f  x 
x
f  x   4  x dx  3  x 4 dx
3

 x 4   x 3 
f  x   4    3 C
  
4  3 

Class XII Maths www.vedantu.com 7


1
f  x   x4  C
x3
Also,
f  2  0
1
f  2   2  3 C  0
4

 
2
1
 16   C  0
8
129
C
8
1 129
f  x   x4  3 
x 8
Thus, the correct answer is A.

Exercise 7.2

2x
1.
1  x2
Ans: Substitute 1  x 2  t
 2xdx  dt
2x 1
 dx   dt
1  x2 t
 log t  C
 log 1  x 2  C
 log 1  x 2   C

 log x 
2

2.
x
Ans: Substitute log x  t
1
 dx  dt
x
 log x  dx 
2

  t dt
2

Class XII Maths www.vedantu.com 8


t3
 C
3
 log x 
3

 C
3

1
3.
x  xlog x
1 1
Ans: 
x  x log x x 1  log x 
Substitute 1  log x  t
1
 dx  dt
x
1 1
 dx   dt
x 1  log x  t
 log t  C
 log 1  log x  C

4. Sin x.sin  cos x 


Ans: Sin x.sin  cos x 
Put,cos x  t
 sin xdx  dt
  sin x.sin  cos x  dx    sin tdt
    cos t   C
 cos t  C
 cos  cos x   C

5. Sin  ax  b  cos  ax  b 
2sin  ax  b  cos  ax  b  sin 2  ax  b 
Ans: Sin  ax  b  cos  ax  b   
2 2
Substitute 2  ax  b   t
 2adx  dt
sin 2  ax  b  1 sin t
 dx   dt
2 2 2a

Class XII Maths www.vedantu.com 9


1
   cos t   C
4a
1
 cos 2  ax  b   C
4a

6. ax  b
Ans: Substitute ax  b  t
 adx  dt
1
 dx  dt
a
1
1 1
   ax  b  2 dx   t 2 dt
a
 1
1  t2  2 3
    C   ax  b  2  C
a 3  3a
2

7. x x  2
Ans: Substitute x  2  t
 dx  dt
  x x  2    t  2  tdt
 3 1

   t 2  2t 2 dt
 
3 1

  t dt  2  t dt
2 2

2 52 4 32
 t  t C
5 3

8. x 1  2x 2
Ans: Substitute 1  2x 2  t
 4xdx  dt
tdt
  x 1  2x 2 dx  
4
1 12
  t dt
4

Class XII Maths www.vedantu.com 10


 3
1  t2 
  C
4 3 
2
 1  2x   C
3
1 2 2

9.  4x  2  x 2  x  1
Ans: Substitute x 2  x  1  t
  2x  1 dx  dt

  4x  2  x 2  x  1dx
  2 tdt
 2  tdt
 3 
 t2 
 
3
4 2
 2   C  x  x  1 2
C
3 3
 
2 

1
10.
x x
1 1
Ans: 
x x x x 1  
Substitute  
x 1  t
1 2
 dx   dt
x  x 1  t

 2log t  C
 2log x  1  C

x
11. ,x  0
x4
Ans: Substitute x  4  t

Class XII Maths www.vedantu.com 11


 dx  dt
x  t  4  dt   t  4 dt
 x4
dx    
t  t
3  1
t 2  t2  2 32 1
  4   C   t   8 t 2  C
3 1 3
 
2 2
1 1
2 2
 t.t  8t 2  C
3
2 1
 t 2  t  12   C
3
2
 x  4  x  8  C
3

12.  x  1 x 5
1
3 3

Ans: Substitute x 3  1  t
 3x 2dx  dt

   x  1 x dx    x  1 x 3 .x 2dx
1 1
3 3 5 3 3

1
dt 1  43 13 
  t 3  t  1   t  t  dt
3 3  
1  3 73 3 34 
  t  t C
3 7 4 

  x  1   x  1  C
7 4
1 3 3
1 3 3

7 4

x2
13.
 2  3x  3 3

Ans: Substitute 2  3x 3  t
 9x 2dx  dt
x2 1 dt
 dx   3
 2  3x  3 3
9 t

Class XII Maths www.vedantu.com 12


1  t 2 
  C
9  2 
1
 C
18  2  3x 
3 2

1
14. ,x  0
x  log x 
m

Ans: Substitute log x  t


1
dx  dt
x
1 dt  t  m 1 
 m dx   m  C
x  log x   t   1  m 
 log x 
1 m

 C
1  m 
x
15.
9  4x 2
Ans: Substitute 9  4x 2  t
8xdx  dt
x 1 1
 dx   dt
9  4x 2
8 t
1
 log t  C
8
1
 log 9  4x 2  C
8

16. e 2x 3
Ans: Substitute 2x  3  t
 2dx  dt
1
  e 2 x 3dx   e t dt
2
  et   C
1
2
1
 e  2 x  3  C
2
Class XII Maths www.vedantu.com 13
x
17. 2
ex
Ans: Substitute x 2  t
 2xdx  dt
x 1 1
  x dx   t dt 2
e 2 e
1
  e  t dt
2
1  e t 
  C
2  1 
1
  e x  C
2

2
1
 x C 2
2e
1
etan x
18.
1  x2
Ans: Substitute tan 1 x  t
1
 dx  dt
1  x2
1
e tan x
 dx  dt
1  x2
 et  C
1
 e tan x
C

e 2x  e 2x
19. Solve the following: 2x .
e  e 2x
e 2 x  e 2 x
Ans: Given expression 2 x .
e  e2 x
Let us substitute e2x  e2x  t , we get
 2e2x  2e2x  dx  dt
 2  e2x  e2x  dx  dt
Integration of given expression is
e2x  e2x dt
  2x 
e e 2x
2t

Class XII Maths www.vedantu.com 14


e2x  e2x 1 1
  2x   dt
e  e2x 2 t
e2x  e2x 1
  2x  log t  C
e  e2x 2
Again substitute t  e2x  e2x , we get
e2x  e2x 1
  2x  log e2x  e2x  C
e e 2x
2

20. Solve the following: tan 2  2x  3  .


Ans: Given expression tan 2  2x  3 .
We can apply the identity tan 2 x  sec2 x  1 , we get
tan 2  2x  3  sec 2  2x  3  1
Substitute 2x  3  t , we get
2dx  dt
Integration of given expression is
  tan 2  2x  3 dx   sec2  2x  3  1dx
1
  tan 2  2x  3 dx   sec 2 tdt  1dx
2
1
  tan 2  2x  3 dx  tan t  x  C
2
Substitute 2x  3  t
1
  tan 2  2x  3 dx  tan  2x  3  x  C
2

21. Solve the following: sec2  7  4x  .


Ans: Given expression sec2  7  4x  .
Put 7  4x  t , we get
4dx  dt
Integration of given expression is
1
  sec2  7  4x  dx    sec 2 tdt
4
1
  sec2  7  4x  dx   tan t  C
4
Substitute 7  4x  t , we get
1
  sec2  7  4x  dx   tan  7  4x   C
4

Class XII Maths www.vedantu.com 15


sin 1 x
22. Solve the following: .
1  x2
sin 1 x
Ans: Given expression .
1  x2
Put sin 1 x  t , we get
1
 dx  dt
1  x2
Integration of given expression is
sin 1 x
 dx   tdt
1 x 2

sin 1 x t2
 dx   C
1  x2 2
Substitute sin x  t , we get
1

 sin 1 x 
2
sin 1 x
 dx  C
1  x2 2

2cos x  3sin x
23. Solve the following: .
6cos x  4sin x
2cos x  3sin x
Ans: Given expression is .
6cos x  4sin x
Given expression can be written as
2cos x  3sin x 2cos x  3sin x

6cos x  4sin x 2  3cos x  2sin x 
Let 3cos x  2sin x  t , we get
 3sin x  2cos x  dx  dt
Integration of given expression is
2cos x  3sin x 2cos x  3sin x
 dx   dx
6cos x  4sin x 2  3cos x  2sin x 
2cos x  3sin x dt
 dx  
6cos x  4sin x 2t
2cos x  3sin x 1 1
 dx   dt
6cos x  4sin x 2 t
2cos x  3sin x 1
 dx  log t  C
6cos x  4sin x 2
Substitute 3cos x  2sin x  t

Class XII Maths www.vedantu.com 16


2cos x  3sin x 1
 dx  log 2sin x  3cos x  C
6cos x  4sin x 2

1
24. Solve the following: .
cos x  1  tan x 
2 2

1
Ans: Given expression 2 .
cos 2 x 1  tan x 
Given expression can be written as
1 sec 2 x
2 
cos 2 x 1  tan x  1  tan x 
2

Let 1  tan x   t , we get


 sec2 xdx  dt
Integration of given expression is
1 sec 2 x
 2 dx   2 dx
cos 2 x 1  tan x  1  tan x 
1 dt
 dx  
cos 2 x 1  tan x 
2
t2
1
 2 dx    t dt
2

cos x 1  tan x 
2

1 1
 2 dx  C
cos x 1  tan x 
2
t
Substitute 1  tan x   t ,
1 1
 dx  C
cos x 1  tan x 
2 2
1  tan x 

cos x
25. Solve the following: .
x
cos x
Ans: Given expression is .
x
Let x  t , we get
1
dx  dt
2 x
Integration of given expression is

Class XII Maths www.vedantu.com 17


cos x
 dx  2 cos tdt
x
cos x
 dx  2sin t  C
x
Substitute x  t
cos x
 dx  2sin x  C
x

26. Solve the following: sin 2x cos 2x .


Ans: Given expression is sin 2x cos 2x .
Let sin 2x  t , we get
2cos2xdx  dt
Integration of given expression is
1
  sin 2x cos 2xdx   tdt
2
 3
1  t2 
  sin 2x cos 2xdx     C
2 3 
2
1 3
  sin 2x cos 2xdx  t 2  C
3
Substitute sin 2x  t
1 3
  sin 2x cos 2xdx   sin 2x  2  C
3

cos x
27. Solve the following: .
1  sin x
cos x
Ans: Given expression .
1  sin x
Let 1  sin x  t
cos xdx  dt
Integration of given expression is
cos x dt
 dx  
1  sin x t

Class XII Maths www.vedantu.com 18


1

cos x t 2
 dx   C
1  sin x 1
2
cos x
 dx  2 t  C
1  sin x
Substitute 1  sin x  t ,
cos x
 dx  2 1  sin x  C
1  sin x

28. Solve the following: cot xlogsin x .


Ans: Given expression cot x logsin x .
Let logsin x  t , we get
1
cos xdx  dt
sin x
 cot xdx  dt
Integration of given expression is
 cot x logsin xdx   tdt
t2
  cot x logsin xdx  C
2
Substitute logsin x  t ,
1
  cot x logsin xdx   logsin x   C
2

sin x
29. Solve the following: .
1  cos x
sin x
Ans: Given expression .
1  cos x
Let 1  cos x  t
 sin xdx  dt
Integration of given expression is
sin x dt
 dx   
1  cos x t
sin x
 dx   log t  C
1  cos x
Substitute 1  cos x  t ,
sin x
 dx   log 1  cos x  C
1  cos x
Class XII Maths www.vedantu.com 19
sin x
30. Solve the following: .
 1  cos x 
2

sin x
Ans: Given expression 2 .
1  cos x 
Let 1  cos x  t
 sin xdx  dt
Integration of given expression is
sin x dt
 2 dx    2
1  cos x  t
sin x
 2 dx    t dt
2

1  cos x 
sin x 1
 2 dx  C
1  cos x  t
Substitute 1  cos x  t ,
sin x 1
 2 dx  C
1  cos x  1  cos x

1
31. Solve the following: .
1  cot x
1
Ans: Given expression .
1  cot x
1
Let I   dx
1  cot x
Integration of given expression is
1
I dx
1  cot x
1
I dx
cos x
1
sin x
sin x
I dx
sin x  cos x
1 2sin x
I  dx
2 sin x  cos x
1  sin x  cos x    sin x  cos x 
I  dx
2 sin x  cos x

Class XII Maths www.vedantu.com 20


1  sin x  cos x   sin x  cos x 
I 
2 sin x  cos x

sin x  cos x
dx

1 1  sin x  cos x 
 I  1dx   dx
2 2 sin x  cos x
Let sin x  cos x  t
  cos x  sin x  dx  dt
Substitute in above obtained equation, we get
1 1 dt
 I  x  
2 2 t
x 1
 I   log t  C
2 2
Substitute sin x  cos x  t ,
x 1
 I   log sin x  cos x  C
2 2

1
32. Solve the following: .
1  tan x
1
Ans: Given expression .
1  tan x
1
Let I   dx
1  tan x
Integration of given expression is
1
I dx
1  tan x
1
I dx
sin x
1
cos x
cos x
I dx
cos x  sin x
1 2cos x
I  dx
2 cos x  sin x
1  cos x  sin x    cos x  sin x 
I  dx
2 cos x  sin x
1  cos x  sin x   cos x  sin x 
I   dx
2 cos x  sin x cos x  sin x
1 1  cos x  sin x 
 I  1dx   dx
2 2 cos x  sin x
Class XII Maths www.vedantu.com 21
Let cos x  sin x  t
   sin x  cos x  dx  dt
Substitute in above obtained equation, we get
1 1 dt
 I  x  
2 2 t
x 1
 I   log t  C
2 2
Substitute cos x  sin x  t ,
x 1
 I   log cos x  sin x  C
2 2

tan x
33. Solve the following: .
sin xcos x
tan x
Ans: Given expression .
sin x cos x
tan x
Let I   dx
sin x cos x
Multiply and divide by cos x , we get
tan x  cos x
I dx
sin x cos x  cos x
tan x  cos x
I dx
tan x  cos 2 x
sec 2 x
I dx
tan x
Let tan x  t
sec2 xdx  dt
Substitute in above obtained equation, we get
dt
 I   dx
t
I2 t C
Substitute tan x  t ,
 I  2 tan x  C

1  log x 
2

34. Solve the following: .


x

Class XII Maths www.vedantu.com 22


1  log x 
2

Ans: Given expression .


x
Let 1  log x  t
1
 dx  dt
x
Integration of given expression is
1  log x  dx  t 2dt
2

 x

1  log x 
2
t3
 dx   C
x 3
Substitute 1  log x  t
1  log x  1  log x 
2 3

 dx  C
x 3

 x  1 x  log x 
2

35. Solve the following: .


x
 x  1 x  log x 
2

Ans: Given expression .


x
Given expression cab be written as
 x  1 x  log x    x  1  x  log x 2
2

  
x  x 
 x  1 x  log x 
2
 1
  1    x  log x 
2

x  x
Let x  log x  t
 1
 1   dx  dt
 x
Integration of given expression is
 x  1 x  log x 
2

 dx   t 2dt
x
 x  1 x  log x 
2
t3
 dx   C
x 3
Substitute x  log x  t
 x  1 x  log x 
2
1
 dx   x  log x   C
3

x 3

Class XII Maths www.vedantu.com 23


x 3 sin  tan 1 x 4 
36. Solve the following: .
1  x8
x 3 sin  tan 1 x 4 
Ans: Given expression .
1  x8
Let x 4  t ,
 4x 3dx  dt
Integration of given expression is
x 3 sin  tan 1 x 4  1 sin  tan t 
1

 dx   dt ………(1)
1  x8 4 1 t2
Let tan 1 t  u
1
 dt  du
1  t2
Substitute in eq. (1), we get
x 3 sin  tan 1 x 4  1
 dx   sin udu
1 x 8
4
x sin  tan x 
3 1 4
1
 dx    cos u   C
1 x 8
4
Substitute tan t  u ,
1

x 3 sin  tan 1 x 4 
dx   cos  tan 1 t   C
1

1 x 8
4
Substitute x  t ,
4

x 3 sin  tan 1 x 4 
dx   cos  tan 1 x 4   C
1

1 x 8
4

10x9  10x log e 10


37.  dx equals
x10  10x
(A) 10x  x10  C
(B) 10x  x10  C
(C)  10x  x10   C
1

(D) log  10x  x10   C


10x 9  10x log e 10
Ans: Given expression  dx .
x10  10x
Let x10  10x  t ,
10x 9  10x loge 10  dx  dt
Integration of given expression is

Class XII Maths www.vedantu.com 24


10x 9  10x log e 10 dt
 dx  
x  10
10 x
t
10x  10 log e 10
9 x

 dx  log t  C
x10  10x
Substitute x10  10x  t ,
10x 9  10x log e 10
 dx  log 10x  x10   C
x  10
10 x

Therefore, option D is the correct answer.

dx
38.  sin 2
cos 2 x
equals
(A) tanx  cot x  C
(B) tanx  cot x  C
(C) tanxcot x  C
(D) tanx  cot 2x  C
dx
Ans: Given expression  sin 2 x cos2 x .
dx
Let I  
sin 2 x cos 2 x
1
I 2 dx
sin x cos 2 x
We know that sin 2 x  cos 2 x  1 , we get
sin 2 x  cos2 x
I dx
sin 2 x cos 2 x
sin 2 x cos 2 x
I 2 dx   sin 2 x cos 2 x dx
sin x cos 2 x
1 1
I 2
dx   2 dx
cos x sin x
 I   sec xdx   cosec2 xdx
2

 I  tan x  cot x  C
Therefore, option B is the correct answer.

Exercise 7.3

1. Solve the following: sin 2  2x  5  .


Ans: Given expression sin 2  2x  5  .
Given expression can be written as

Class XII Maths www.vedantu.com 25


1  cos 2  2x  5
sin 2  2x  5 
2
1  cos  4x  10 
 sin 2  2x  5 
2
Integration of given expression is
1  cos  4x  10 
  sin 2  2x  5 dx   dx
2
1 1
  sin 2  2x  5  dx  1dx   cos  4x  10  dx
2 2
1 1  sin  4x  10  
  sin 2  2x  5  dx  x   C
2 2 4 
1 1
  sin 2  2x  5  dx  x  sin  4x  10   C
2 8

2. Solve the following: sin3xcos4x .


Ans: Given expression sin3xcos4x .
Using the identity sin Acos B  sin  A  B   sin  A  B  given expression
1
2
can be written as
sin 3x cos 4x  sin  3x  4x   sin  3x  4x 
1
2
Integration of above expression is
  sin 3x cos 4xdx   sin 7x  sin   x  dx
1
2
1
  sin 3x cos 4xdx   sin 7x  sin x dx
2
1 1
  sin 3x cos 4xdx   sin 7xdx   sin xdx
2 2
1   cos7x  1
  sin 3x cos 4xdx       cos x   C
2 7  2
 cos7x cos x
  sin 3x cos 4xdx   C
14 2

3. Solve the following: cos2xcos4xcos6x .


Ans: Given expression cos2xcos4xcos6x .

Class XII Maths www.vedantu.com 26


Using the identity cos A cos B 
1
2
cos  A  B  cos  A  B given expression
can be written as
cos 2x  cos 4x cos6x   cos 2x
1
2
cos  4x  6x   cos  4x  6x 
Integration of the above expression is
1 
  cos 2x  cos 4x cos6x    cos 2x  cos10x  cos  2x  dx
2 
1 
  cos 2x  cos 4x cos6x     cos 2x cos10x  cos 2x cos  2x dx
2 
  cos 2x  cos 4x cos6x    cos 2x cos10x  cos 2 2xdx
1
2
Again applying the identity cos A cos B  cos  A  B   cos  A  B  , we get
1
2
 1 
 cos  2x  10x   cos  2x  10x  
1   2 

  cos 2x  cos 4x cos6x      dx
2    1  cos 4x  
  
  2  

1
  cos 2x  cos 4x cos6x    cos12x  cos8x  cos 4x dx
4
1  sin12x sin8x sin 4x 
  cos 2x cos 4x cos6x     C
4  12 8 4 

4. Solve the following: sin 3  2x  1 .


Ans: Given expression sin 3  2x  1 .
Let I   sin 3  2x  1 dx
 I   sin 2  2x  1 sin  2x  1 dx
 I   1  cos 2  2x  1  sin  2x  1 dx
Let cos  2x  1  t
2sin  2x  1 dx  dt
Integration becomes
 I    1  t 2  dt
1
2
1  t3 
 I   t  C
2 3
Class XII Maths www.vedantu.com 27
Substitute cos  2x  1  t ,
1 cos3  2x  1 
 I    cos  2x  1  C
2 3 
 cos  2x  1 cos3  2x  1
  sin  2x  1 dx 
3
 C
2 3

5. Solve the following: sin3 xcos 3 x .


Ans: Given expression sin 3 x cos3 x .
Let I   sin 3 x cos3 xdx
 I   sin 2 xsin x cos3 xdx
 I   cos3 x 1  cos2 x  sin xdx
Let cos x  t
 sindx  dt
Integration becomes
 I   t 3 1  t 2 dt
 I     t 3  t 5 dt
 t4 t6 
 I      C
4 6
Substitute cos x  t ,
 cos 4 x cos6 x 
 I    C
 4 6 
cos6 x cos 4 x
  sin x cos xdx 
3 3
 C
6 4

6. Solve the following: sinxsin2xsin3x .


Ans: Given expression sin xsin 2xsin3x .
1
Using the identity sin Asin B  cos  A  B   cos  A  B  , given expression can
2
be written as
1
 sin x sin 2x sin 3x  sin x. cos  2x  3x   cos  2x  3x 
2
Integration of given expression is
  sin x sin 2x sin 3xdx    sin x cos  x   sin x cos  5x   dx
1
2

Class XII Maths www.vedantu.com 28


1
  sin x sin 2x sin 3xdx 
2
 sin x cos x  sin x cos5x  dx
1 sin 2x 1
  sin x sin 2x sin 3xdx   dx    sin x cos5x dx
2 2 2
1   cos 2x  1  1 
  sin x sin 2x sin 3xdx        sin  x  5x   sin  x  5x    dx
4 2  2 2 
 cos 2x 1
  sin x sin 2x sin 3xdx 
8 4
 
   sin 6x  sin  4x   dx
 cos 2x 1
  sin x sin 2x sin 3xdx     sin 6x  sin 4x  dx
8 4
 cos 2x 1   cos6x cos 4x 
  sin x sin 2x sin 3xdx     C
8 8 3 4 
1  cos6x cos 4x 
  sin x sin 2x sin 3xdx     cos 2x   C
8 3 4 

7. Solve the following: sin4xsin8x .


Ans: Given expression sin 4xsin8x .
1
Using the identity sin Asin B  cos  A  B   cos  A  B  , given expression can
2
be written as
1
 sin 4x sin8x  cos  4x  8x   cos  4x  8x 
2
Integration of given expression is
  sin 4x sin8xdx    cos  4x   cos 12x   dx
1
2
1
  sin 4x sin8xdx    cos 4x  cos12x  dx
2
1  sin 4x sin12x 
  sin 4x sin8xdx    C
2 4 12 

1  cos x
8. Solve the following: .
1  cos x
1  cos x
Ans: Given expression .
1  cos x
x x
Using the identities 2sin 2  1  cos x and cos x  2cos 2  1 given expression
2 2
can be written as

Class XII Maths www.vedantu.com 29


x
1  cos x 2sin 2
  2
1  cos x 2cos 2 x
2
1  cos x x
  tan 2
1  cos x 2
Integration of given expression is
1  cos x  x
 dx    tan 2  dx
1  cos x  2
1  cos x  x 
 dx   sec 2  1 dx
1  cos x  2 
 x 
 tan

1  cos x
dx   2  x  C

1  cos x 
1

 2 
1  cos x x
 dx  2 tan  x  C
1  cos x 2

cos x
9. Solve the following: .
1  cos x
cos x
Ans: Given expression .
1  cos x
x x x
Using the identity cos x  cos 2  sin 2 and cos x  2cos 2  1 given
2 2 2
expression can be written as
2 x 2 x
cos x cos  sin
  2 2
1  cos x 2cos 2
x
2
 2 x 
1 sin

cos x
 1  2

1  cos x 2  cos 2 x 
 2
cos x 1 x
  1  tan 2 
1  cos x 2  2
Integration of given expression is

Class XII Maths www.vedantu.com 30


cos x 1  x
 dx   1  tan 2  dx
1  cos x 2  2
cos x 1  x 
 dx   1  sec 2  1 dx
1  cos x 2  2 
cos x 1  x
 dx    2  sec 2  dx
1  cos x 2  2
 x
1 tan

cos x
dx   2x  2C
1  cos x 2 1 

 2 
cos x x
 dx  x  tan  C
1  cos x 2

10. Solve the following: sin4 x .


Ans: Given expression sin 4 x .
Given expression can be written as sin 4 x  sin 2 x sin 2 x
 1  cos 2x  1  cos 2x 
 sin 4 x    
 2  2 
1
 sin 4 x  1  cos 2x 
2

4
 sin 4 x  1  cos 2 2x  2cos 2x 
1
4
1  1  cos 4x 
 sin 4 x  1   2cos 2x 
4 2 
1  1 cos 4x 
 sin 4 x  1    2cos 2x 
4 2 2 
1 3 1 
 sin 4 x    cos 4x  2cos 2x 
4 2 2 
Integration of given expression is
1 3 1 
  sin 4 xdx     cos 4x  2cos 2x dx
4 2 2 
1 3 sin 4x 
  sin 4 xdx   x   sin 2x   C
4 2 8 
3x sin 4x 1
  sin 4 xdx    sin 2x  C
8 32 4

Class XII Maths www.vedantu.com 31


11. Solve the following: cos4 2x .
Ans: Given expression cos 4 2x .
Given expression can be written as
cos 4 2x   cos 2 2x 
2

 1  cos 4x 
2

 cos 2x  
4

 2 
 cos 4 2x  1  cos 2 4x  2cos 4x 
1
4
1  1  cos8x 
 cos 4 2x  1   2cos 4x 
4 2 
1  1 cos8x 
 cos 4 2x  1    2cos 4x 
4 2 2 
1  3 cos8x 
 cos 4 2x     2cos 4x 
4 2 2 
Integration of given expression is
 3 cos8x cos 4x 
  cos 4 2xdx       dx
8 8 2 
3 sin8x sin 4x
  cos 4 2xdx  x   C
8 64 8

sin 2 x
12. Solve the following: .
1  cos x
sin 2 x
Ans: Given expression .
1  cos x
x x x
By applying the identity sin x  2sin cos and cos x  2cos 2  1 , given
2 2 2
expression can be written as
2
 x x
 2sin cos 

sin 2 x 2 2

1  cos x 2cos 2
x
2
2 x 2 x
2
sin x 4sin cos
  2 2
1  cos x 2cos 2
x
2

Class XII Maths www.vedantu.com 32


sin 2 x x
  2sin 2
1  cos x 2
2
sin x
  1  cos x
1  cos x
Integration of given expression is
sin 2 x
 dx  1dx   cos xdx
1  cos x
sin 2 x
 dx  x  sin x  C
1  cos x

cos 2x  cos 2
13. Solve the following: .
cos x  cos 
cos 2x  cos 2
Ans: Given expression .
cos x  cos 
CD CD
We can apply the identity cosC  cos D  2sin sin , we get
2 2
2x  2 2x  2
cos 2x  cos 2 2sin sin
  2 2
cos x  cos  x   x  
2sin sin
2 2
2 x   2 x  
cos 2x  cos 2 sin sin
  2 2
cos x  cos  x x
sin sin
2 2
cos 2x  cos 2 sin  x    sin  x   
 
cos x  cos  x x
sin sin
2 2
We can apply the identity sin 2x  2sin xcos x , we get
 x x   x x 
2sin cos 2sin cos
cos 2x  cos 2  2 2   2 2 
 
cos x  cos  x x
sin sin
2 2
cos 2x  cos 2 x x
  4cos cos
cos x  cos  2 2
cos 2x  cos 2  x  x  x   x 
  2 cos   cos 
cos x  cos   2 2 2 2 

Class XII Maths www.vedantu.com 33


cos 2x  cos 2
  2cos x  cos  
cos x  cos 
Integration of given expression is
cos 2x  cos 2
 dx  2 cos x  cos  dx
cos x  cos 
cos 2x  cos 2
 dx  2sin x  x cos    C
cos x  cos 

cos x  sin x
14. Solve the following: .
1  sin 2x
cos x  sin x
Ans: Given expression .
1  sin 2x
We know that sin 2 x  cos 2 x  1 .
Given expression can be written as
cos x  sin x cos x  sin x
  2 .
1  sin 2x sin x  cos 2 x  sin 2x
We can apply the identity sin 2x  2sin xcos x , we get
cos x  sin x cos x  sin x
  2
1  sin 2x sin x  cos 2 x  2sin x cos x
cos x  sin x cos x  sin x
 
1  sin 2x  sin x  cos x 
2

Let sin x  cos x  t


  cos x  sin x  dx  dt
Integration of given expression is
cos x  sin x cos x  sin x
 dx   2 dx
1  sin 2x  sin x  cos x 
cos x  sin x dt
 dx   2
1  sin 2x t
cos x  sin x
 dx   t 2dt
1  sin 2x
cos x  sin x
 dx   t 1  C
1  sin 2x
cos x  sin x 1
 dx    C
1  sin 2x t
Substitute sin x  cos x  t ,
cos x  sin x 1
 dx   C
1  sin 2x sin x  cos x

Class XII Maths www.vedantu.com 34


15. Solve the following: tan 3 2xsec2x .
Ans: Given expression tan 3 2x sec2x .
Given expression can be written as
tan 3 2x sec2x  tan 2 2x tan 2x sec2x
 tan 3 2xsec2x   sec2 2x  1 tan 2xsec2x
 tan 3 2x sec2x  sec2 2x tan 2x sec2x  tan 2x sec2x
Integration of given expression is
  tan 3 2xsec2xdx   sec2 2x tan 2xsec2xdx   tan 2xsec2xdx
sec 2x
  tan 3 2x sec 2xdx   sec 2 2x tan 2x sec 2xdx  C
2
Let sec2x  t
2sec2x tan 2xdx  dt
Above integral becomes
1 sec 2x
  tan 3 2x sec 2xdx   t 2dt  C
2 2
t 3 sec2x
  tan 3 2x sec2xdx   C
6 2
Substitute sec2x  t ,
 sec 2x  sec 2x
3

  tan 2x sec 2xdx 


3
 C
6 2

16. Solve the following: tan 4 x .


Ans: Given expression tan 4 x .
Given expression can be written as
 tan 4 x  tan 2 x tan 2 x
 tan 4 x   sec2 x  1 tan 2 x
 tan 4 x  sec2 x tan 2 x  tan 2 x
 tan 4 x  sec2 x tan 2 x   sec2 x  1
 tan 4 x  sec2 x tan 2 x  sec2 x  1
Integration of given expression is
  tan 4 xdx    sec2 x tan 2 x  sec2 x  1dx
  tan 4 xdx    sec2 x tan 2 x dx   sec2 xdx  1dx
  tan 4 xdx   sec2 x tan 2 xdx  tan x  x  C
Let tan x  t
sec2 xdx  dt

Class XII Maths www.vedantu.com 35


  tan 4 xdx   t 2dt  tan x  x  C
t3
  tan xdx   tan x  x  C
4

3
Substitute tan x  t ,
1
  tan 4 xdx  tan 3 x  tan x  x  C
3

sin 3 x  cos 3 x
17. Solve the following: .
sin 2 xcos 2 x
sin 3 x  cos3 x
Ans: Given expression .
sin 2 x cos 2 x
Given expression can be written as
sin 3 x  cos3 x sin 3 x cos3 x
  
sin 2 x cos2 x sin 2 x cos2 x sin 2 x cos2 x
sin 3 x  cos3 x sin x cos x
  
sin 2 x cos 2 x cos 2 x sin 2 x
sin 3 x  cos3 x
  tan xsec x  cot x cosecx
sin 2 x cos 2 x
Integration of given expression is
sin 3 x  cos3 x
 dx   tan xsec xdx   cot x cosecx dx
sin 2 x cos 2 x
sin 3 x  cos3 x
 dx  sec x  cosecx  C
sin 2 x cos 2 x

cos 2x  2sin 2 x
18. Solve the following: .
cos 2 x
cos 2x  2sin 2 x
Ans: Given expression .
cos 2 x
By applying the identity cos 2x  1  2sin 2 x , we get
cos 2x  2sin 2 x cos 2x  1  cos 2x
 
cos 2 x cos 2 x
cos 2x  2sin 2 x 1
 2

cos x cos 2 x
cos 2x  2sin 2 x
 2
 sec2 x
cos x
Integration of given expression is

Class XII Maths www.vedantu.com 36


cos 2x  2sin 2 x
 2
dx   sec2 xdx
cos x
cos 2x  2sin 2 x
 dx  tan x  C
cos 2 x

1
19. Solve the following: .
sin xcos 3 x
1
Ans: Given expression .
sin x cos3 x
We can apply the identity sin 2 x  cos 2 x  1 , we get
1 sin 2 x  cos 2 x
 
sin x cos3 x sin x cos3 x
1 sin 2 x cos 2 x
  
sin x cos3 x sin x cos3 x sin x cos 3 x
1 sin x 1
 3
 3

sin x cos x cos x sin x cos x
1 cos 2 x
  tan x sec x 
2

sin x cos3 x sin x cos x


cos 2 x
1 sec2 x
  tan x sec 2
x 
sin x cos3 x tan x
Integration of given expression is
1 sec2 x
 dx   tan xsec xdx  
2
dx
sin x cos3 x tan x
Let tan x  t
sec2 xdx  dt
1 1
 3
dx   tdt   dt
sin x cos x t
2
1 t
 3
dx   log t  C
sin x cos x 2
Substitute tan x  t ,
1 1
 3
dx  tan 2 x  log tan x  C
sin x cos x 2

cos 2x
20. Solve the following: .
 cos x  sin x 
2

Class XII Maths www.vedantu.com 37


cos 2x
Ans: Given expression 2 .
 cos x  sin x 
Given expression can be written as
cos 2x cos 2x
 2 
 cos x  sin x  cos x  sin 2 x  2sin x cos x
2

We know that sin 2 x  cos 2 x  1 and 2sin xcos x  sin 2x , we get


cos 2x cos 2x
 2 
 cos x  sin x  1  sin 2x
Integration of given expression is
cos 2x cos 2x
 2 dx   dx
 cos x  sin x  1  sin 2x
Let 1  sin 2x  t
2cos2xdx  dt
Integration becomes
cos 2x 1 1
 2 dx   t dt
 cos x  sin x  2
cos 2x 1
 2 dx  log t  C
 cos x  sin x  2
Substitute 1  sin 2x  t
cos 2x 1
 2 dx  log 1  sin 2x  C
 cos x  sin x  2
cos 2x 1
 2 dx  log  cos x  sin x   C
2

 cos x  sin x  2
cos 2x
 2 dx  log  cos x  sin x   C
 cos x  sin x 
21. Solve the following: sin 1  cos x  .
Ans: Given expression sin 1  cos x  .
Let cos x  t
 sin x  1  t 2
  sin xdx  dt
dt
 dx  
sin x
dt
 dx  
1  t2
Class XII Maths www.vedantu.com 38
Integration of given expression is
 dt 
  sin 1  cos x  dx   sin 1 t  2 
 1 t 
 sin 1 t 
  sin  cos x  dx    
1
2 
dt
 1  t 
Let sin t  u
1

1
 dt  du
1  t2
Integration becomes
  sin 1  cos x  dx   4du
u2
  sin  cos x  dx    C
1

2
Substitute sin t  u
1

 sin 1 t 
2

  sin  cos x  dx  
1
C
2
Substitute cos x  t
 
2
 sin 1
cos x 
  sin 1  cos x  dx      C ……..(1)
2

We know that sin 1 x  cos 1 x 
2
  
 sin 1  cos x    cos 1  cos x     x 
2 2 
Substitute in eq. (1), we get

2

  x 
  sin  cos x  dx    C
1 2
2
1  2

  sin 1  cos x  dx    x 2  x   C
2 2 
 x 1
2 2

  sin 1  cos x  dx     x  C
4 2 2
x x 2  2 
  sin 1  cos x  dx    C  
2 2  4
x x 2
  sin 1  cos x  dx    C1
2 2

Class XII Maths www.vedantu.com 39


1
22. Solve the following: .
cos  x  a  cos  x  b 
1
Ans: Given expression .
cos  x  a  cos  x  b 
Given expression can be written as
1 1  sin  a  b  
   
cos  x  a  cos  x  b  sin  a  b   cos  x  a  cos  x  b  
1 1  sin  x  b    x  a   
   
cos  x  a  cos  x  b  sin  a  b   cos  x  a  cos  x  b  
1 1  sin  x  b  cos  x  a   cos  x  b  sin  x  a  
   
cos  x  a  cos  x  b  sin  a  b   cos  x  a  cos  x  b  
1 1
   tan  x  b   tan  x  b  
cos  x  a  cos  x  b  sin  a  b  
Integration of given expression is
1 1
 dx    tan  x  b   tan  x  b   dx
cos  x  a  cos  x  b  sin  a  b  
1 1
 dx     log cos  x  b   log cos  x  a  
cos  x  a  cos  x  b  sin  a  b  
1 1  cos  x  a  
 dx    log C
cos  x  a  cos  x  b  sin  a  b   cos  x  b  

sin 2 x  cos 2 x
23. Solve the following:  dx is equal to
sin 2 xcos 2 x
(A) tanx  cot x  C
(B) tanx  cosecx C
(C)  tanx  cot x  C
(D) tanx  secx  C
sin 2 x  cos 2 x
Ans: Given expression  dx .
sin 2 x cos 2 x
Given expression can be written as
sin 2 x  cos 2 x sin 2 x cos 2 x
 dx   2 dx   2 dx
sin 2 x cos 2 x sin x cos 2 x sin x cos 2 x
sin 2 x  cos 2 x
 2 2
dx   sec2 xdx   cosec2 xdx
sin x cos x

Class XII Maths www.vedantu.com 40


sin 2 x  cos 2 x
 dx  tan x  cot x  C
sin 2 x cos 2 x
Therefore, option A is the correct answer.

ex 1  x 
24. Solve the following:  dx equals
cos 2  e x x 
(A)  cot  exx   C
(B) tan  xex   C
(C) tan  ex   C
(D) cot  ex   C
e x 1  x 
Ans: Given expression  dx .
cos 2  e x x 
Let e x x  t
 ex .x  ex .1 dx  dt
 e x  x  1 dx  dt
Integration of given expression is
e x 1  x  dt
 dx  
cos  e x 
2 x
cos 2 t
e x 1  x 
 dx   sec 2 tdt
cos  e x 
2 x

e x 1  x 
 dx  tan t  C
cos 2  e x x 
Substitute e x x  t ,
e x 1  x 
 dx  tan  e x x   C
cos  e x 
2 x

Therefore, option B is the correct answer.

Exercise 7.4
3x 2
1. Solve the following: 6 .
x 1
3x 2
Ans: Given expression 6 .
x 1
Let x 3  t
3x 2dx  dt
Class XII Maths www.vedantu.com 41
Integration of given expression is
3x 2 dt
 6 dx   2
x 1 t 1
1
We know that   tan 1 x
1 x 2

2
3x
 6 dx  tan 1 t  C
x 1
Substitute x 3  t ,
3x 2
  6 dx  tan 1  x 3   C
x 1

1
2. Solve the following: .
1  4x 2
1
Ans: Given expression .
1  4x 2
Let 2x  t
2dx  dt
Integration of given expression is
1 1 dt
 dx  
1  4x 2 2 1  t2
1
We know that  2 dt  log x  x 2  a 2
x a 2

1 1
 dx  log t  t 2  1  C
1  4x 2
2
Substitute 2x  t ,
1 1
 dx  log 2x  4x 2  1  C
1  4x 2
2

1
3. Solve the following: .
 2  x 1
2

1
Ans: Given expression .
2  x 1
2

Let 2  x  t
dx  dt
Integration of given expression is

Class XII Maths www.vedantu.com 42


1 1
 dx    dt
2  x 1 t 1
2 2

1
   
2 2
We know that dt log x x a
x2  a 2

1
 dx   log t  t 2  1  C
2  x 1
2

Substitute 2  x  t ,
1
 dx   log  2  x   2  x 1  C
2

2  x 1
2

1 1
 dx  log C
2  x 1
2
2  x  x 2  4x  5

1
4. Solve the following: .
9  25x 2
1
Ans: Given expression .
9  25x 2
Let 5x  t
5dx  dt
Integration of given expression is
1 1 1
 dx   dt
9  25x 2
5 9  t2
1 1 1
 dx   dt
9  25x 2
5 3 t
2 2

1 1 1  t 
 dx  sin    C
9  25x 2 5 3
Substitute 5x  t ,
1 1 1  5x 
 dx  sin    C
9  25x 2 5  3 

3x
5. Solve the following: .
1  2x 4
3x
Ans: given expression .
1  2x 4
Let 2x 2  t

Class XII Maths www.vedantu.com 43


 2 2dx  dt
Integration of given expression is
3x 3 dt
 dx  
1  2x 4
2 2 1 t
2

3x 3
 dx  tan 1 t  C
1  2x 4
2 2
Substitute 2x  t ,
2


3x
1  2x 4
dx 
3
2 2
 
tan 1 2x 2  C

x2
6. Solve the following: .
1  x6
x2
Ans: Given expression .
1  x6
Let x 3  t
3x 2dx  dt
Integration of given expression is
x2 1 dt
 dx  
1  x6 3 1 t2
x2 1 1 1 t 
 dx   log C
1 x 6
3 2 1  t 
Substitute x 3  t ,
x2 1 1 1  x3 
 dx   log C
1  x6 3 2 1  x3 

x1
7. Solve the following: .
x2  1
x 1
Ans: Given expression .
x2 1
Given expression can be written as
x 1 x 1
  
x2 1 x2 1 x2 1
Integration of given expression is
x 1 x 1
 2 dx   2 dx   2 dx
x 1 x 1 x 1

Class XII Maths www.vedantu.com 44


x 1 x
 dx   dx  log x  x 2  1  C
x 12
x 1
2

Let x  1  t
2

2xdx  dt
Integration becomes
x 1 1 dt
 2 dx    log x  x 2  1  C
x 1 2 t
x 1 1 12
 2 dx   t dt  log x  x 2  1  C
x 1 2
x 1 1  12 
 2 dx   2t   log x  x 2  1  C
x 1 2 
x 1
 2 dx  t  log x  x 2  1  C
x 1
Substitute x 2  1  t
x 1
 2 dx  x 2  1  log x  x 2  1  C
x 1

x2
8. Solve the following: .
x6  a6
x2
Ans: Given expression .
x6  a6
Let x 3  t
3x 2dx  dt
Integration of given expression is
x2 1 dt
 6 dx  
x  a6 3 t 2   a 3 2

x2 1
 dx  log t  t 2  a 6  C
x a
6 6
3
Substitute x  t ,
3

x2 1
 6 dx  log x 3  x 6  a 6  C
x a 6
3

sec2 x
9. Solve the following: .
tan 2 x  4

Class XII Maths www.vedantu.com 45


sec2 x
Ans: Given expression .
tan 2 x  4
Let tan x  t
sec2 xdx  dt
Integration of given expression is
sec 2 x dt
 dx  
tan 2 x  4 t 2  22
sec 2 x
 dx  log t  t 2  4  C
tan x  4
2

Substitute tan x  t ,
sec2 x
 dx  log tan x  tan 2 x  4  C
tan x  4
2

1
10. Solve the following: .
x 2  2x  2
1
Ans: Given expression .
x  2x  2
2

Given expression can be written as


1 1

x 2  2x  2  x  1  1
2 2

Let x  1  t
dx  dt
Integration of given expression is
1 1
 2 dx   2 dt
x  2x  2 t 1
1
 2 dx  log t  t 2  1  C
x  2x  2
Substitute x  1  t ,
1
 2 dx  log  x  1   x  1  1  C
2

x  2x  2
1
 2 dx  log  x  1  x 2  2x  2  C
x  2x  2

1
11. Solve the following: .
9x  6x  5
2

Class XII Maths www.vedantu.com 46


1
Ans: Given expression .
9x  6x  5
2

Given expression can be written as


1 1

9x 2  6x  5  3x  1   2 
2 2

Let 3x  1  t
3dx  dt
Integration of given expression is
1 1 1
 dx   dt
9x  6x  5
2
3 t 2
2 2

1 1  1 1  t  
 dx  tan     C
9x 2  6x  5 3  2  2 
Substitute 3x  1  t ,
1 1  1 1  3x  1  
 dx  tan    C
9x 2  6x  5 3  2  2 

1
12. Solve the following: .
7  6x  x 2
1
Ans: Given expression .
7  6x  x 2
Given expression can be written as
1 1
 
7  6x  x 7   x 2  6x  9  9 
2

1 1
 
7  6x  x 2 16   x 2  6x  9 
1 1
 
7  6x  x 2 16   x  3
2

1 1
 
7  6x  x 2 42   x  3
2

Let x  3  t
dx  dt
Integration of given expression is
1 1
 dx   2 dt
7  6x  x 2 4  t
2

Class XII Maths www.vedantu.com 47


1 1  t 
 dx  sin  C
7  6x  x 2 4
Substitute x  3  t ,
1  x 3
 dx  sin 1  C
7  6x  x 2
 4 
1
13. Solve the following: .
 x  1 x  2 
1
Ans: Given expression .
 x  1 x  2 
Given expression can be written as
1 1
 
 x  1 x  2  x 2  3x  2
1 1
 
 x  1 x  2  x 2  3x 
9 9
 2
4 4
1 1
 
 x  1 x  2  
2
3 1
2

x    
 2 2
3
Let x  t
2
dx  dt
1 1
 dx   dx
 x  1 x  2  1
2

t2   
2
2
1 1
 dx  log t  t     C
2

 x  1 x  2  2
3
Substitute x  t,
2
1  3
 dx  log  x    x 2  3x  2  C
 x  1 x  2   2

1
14. Solve the following: .
8  3x  x 2

Class XII Maths www.vedantu.com 48


1
Ans: Given expression .
8  3x  x 2
Given expression can be written as
1 1
 
8  3x  x 2  9 9
8   x 2  3x   
 4 4
1 1
 
8  3x  x 2  9 9
8   x 2  3x   
 4 4
1 1
 
8  3x  x 2 41  3
2

x  
4  2
3
Let x   t
2
dx  dt
1 1
 dx   dt
8  3x  x 2
 41 
2

  t
2

 2 
 
1  t 
 dx  sin 1
 C
8  3x  x 2  41 
 
 2 
3
Substitute x   t
2
 3
1  x  
 dx  sin 1
 2 C
8  3x  x 2  41 
 
 2 
1 1  2x  3 
 dx  sin  C
8  3x  x 2  41 

1
15. Solve the following: .
 x  a  x  b 

Class XII Maths www.vedantu.com 49


1
Ans: Given expression .
 x  a  x  b 
Given expression can be written as
1 1
 
 x  a  x  b  x 2   a  b  x  ab
1 1
 
 x  a  x  b  a  b
2
a  b
2

x  a  b
2
x   ab
4 4
1 1
 
 x  a  x  b   a  b   a  b
2 2

 
4 
x
 4
1 1
 
 x  a  x  b   a  b   a  b 
2 2

x  4    2 
   
Integration of given expression is
1 1
 dx   dx
 x  a  x  b   a  b   a  b 
2 2

x  4    2 
   
ab
Let x   t
 2 
dx  dt
1 1
 dx   dx
 x  a  x  b  ab
2

t2   
 2 
ab
2
1
 dx  log t  t    C
2

 x  a  x  b   2 
ab
Substitute x   t,
 2 
1 ab
 dx  log x     x  a  x  b  C
 x  a  x  b   2 

Class XII Maths www.vedantu.com 50


1
16.
 x  a  x  b 
Ans: (x  a)(x  b)  x 2  (a  b)x  ab
(a  b) 2 (a  b) 2
x  (a  b)x  ab  x  (a  b)x 
2 2
  ab
4 4
Simplifying,
2
  a  b   (a  b)
2

 x    
  2  4
1 1
 dx   dx
(x  a)(x  b) 2
  a  b   a  b 
2

x      
  2   2 
ab
Consider x     t  dx  dt
 2 
1 1
 dx   dt
ab
2 2
  a  b   a  b 
2

x       t2   
  2   2   2 
Using the logarithm formula of integration,
ab
2

 log t  t  
2
 C
 2 
Substitute the value of t,
  a  b 
 log  x      (x  a)(x  b)  C
  2 

4x  1
17.
2x 2  x  3
 2x 2  x  3  B
d
Ans: Consider 4x  1  A
dx
Simplifying,
 4x  1  A(4x  1)  B
 4x  1  4Ax  A  B
We obtain the below values by equating the coefficients of x and the constant
term on both sides.
4A  4  A 1
A  B 1 B  0

Class XII Maths www.vedantu.com 51


Consider 2x 2  x  3  t
 (4x  1)dx  dt
4x  1 1
 dx   t dt
2x 2  x  3
Using the power rule of integration,
2 t C
Substitute the value of t,
 2 2x 2  x  3  C

x2
18.
x2  1
 x  1  B
d 2
Ans: Consider x  2  A
dx
 x  2  A(2x)  B......1
We obtain the below values by equating the coefficients of x and the constant
term on both sides.
1
2A  1  A 
2
B2
From (1), we get
1
(x  2)  (2x)  2
2
1
x2 (2x)  2
2
 x2 1 dx   x 2  1 dx
1 2x 2
  2 dx   2 dx
2 x 1 x 1
1 2x
In  2 dx let x 2  1  t  2xdx  dt
2 x 1
1 2x 1 dt

2 x2 1
dx  
2 t
Integrating using the power rule
1
 [2 t ]
2
Simplifying,
 t
Substitute the value of t,

Class XII Maths www.vedantu.com 52


 x2 1
2 1
     1
2
Then, dx 2 dx 2log x x
x2 1 x2 1
From equation (2), we get
x2
 x 2  1dx  x  1  2log x  x  1  C
2 2

5x  2
19.
1  2x  3x 2
Ans:
Let 5x  2  A 1  2x  3x 2   B
d
dx
 5 x  2  A  2  6 x   B......1
We obtain the below values by equating the coefficients of x and
the constant term on both sides.
5
5  6A  A 
6
11
2A  B  2  B  
3
Substitute the above values in (1)
5  11 
 5x  2  (2  6x)    
6  3
5 11
(2  6x) 
5x  2
 dx   6 3 dx
1  2x  3x 2
1  2x  3x 2

5 2  6x 11 1
  dx   dx
6 1  2x  3x 2
3 1  2x  3x 2
2  6x 1
Consider I1   dx and I 2   dx
1  2x  3x 2
1  2x  3x 2
5x  2 5 11
 dx  I1  I2 ...1
1  2x  3x 2
6 3
2  6x
I1   dx
1  2x  3x 2
Put 1  2x  3x 2  t
 (2  6x)dx  dt
dt
 I1  
t

Class XII Maths www.vedantu.com 53


Using the logarithm formula of integration,
I1  log | t |
Substitute the value of t,
I1  log 1  2x  3x 2 ... 2 
Then,
1
I2   dx
1  2x  3x 2
 2 
1  2x  3x 2 can be rewritten as 1  3  x 2  x 
 3 
Thus,
 2 
1  3 x 2  x 
 3 
By completing square method,
 2 1 1
 1  3 x 2  x   
 3 9 9
2
 1 1
 1  3 x   
 3 3
Simplifying,
2
2  1
  3 x  
3  3
 1  2
2

 3  x    
 3 9
 1  2  
2 2

 3  x      
  3   3  
Therefore I2 can be rewritten as ,
1 1
I2   dx
3  1  2 
2 2

 x     
3  3  

1 3  3x  1  
  tan 1  
3 2  2 
Simplifying,
1  3x  1 
 tan 1   ... 3
2  2 

Class XII Maths www.vedantu.com 54


We obtain the below values by substituting equations (2) and (3) in equation (1)
5x  2 5 11  1 1  3x  1  
 1  2x  3x 2 dx  6 log 1  2x  3x   3  2 tan  2    C
2

  
Simplifying,
5 11  3x  1 
 log 1  2x  3x 2  tan 1  C
6 3 2  2 

6x  7
20.
 x  5  x  4 
 x  9x  20   B
d 2
Ans: Consider 6x  7  A
dx
Differentiating,
 6 x  7  A  2 x  9  B
We obtain the below values by equating the coefficients of x and the constant
term on both sides.
2A  6  A  3
9 A  B  7  B  34
 6x  7  3(2x  9)  34
6x  7 3(2x  9)  34
 x 2  9x  20   x 2  9x  20 dx
2x  9 1
 3 2 dx  34 2 dx
x  9x  20 x  9x  20
2x  9 1
Consider I1   2 dx and I 2   2 dx
x  9x  20 x  9x  20
6x  7
 2  3I1  34I 2 ....(1)
x  9x  20
2x  9
I1   2 dx
x  9x  20
Put x 2  9x  20  t
 (2x  9)dx  dt
dt
 I1  
t
Integrating using the power rule
I1  2 t
Substitute the value of t,
I1  2 x 2  9x  20 .....(2)
Class XII Maths www.vedantu.com 55
1
I2   dx
x  9x  20
2

Consider
x 2  9x  20
By completing square methods,
81 81
 x 2  9x  20  
4 4
2
 9 1
x   
 2 4
2 2
 9 1
x    
 2 2
1
 I2   2 2 dx
 9 1
x    
 2  2
 9
I2  log  x    x 2  9x  20 ..... 3
 2
We obtain the below values by substituting equations (2) and (3) in (1),
6x  7  9 
 x 2  9x  20 dx  3 2 x  9x  20   34log  x  2   x  9x  20   C
2 2

 
Simplifying,
 9 
 6 x 2  9x  20  34log  x    x 2  9x  20   C
 2 

x2
21.
4x  x 2
 4x  x 2   B
d
Ans: Consider, x  2  A
dx
 x  2  A(4  2x)  B
We obtain the below values by equating the coefficients of x and the
constant term on both sides.
1
2A  1  A  
2
4A  B  2  B  4
1
 (x  2)   (4  2x)  4
2

Class XII Maths www.vedantu.com 56


1
x2  (4  2x)  4
 dx   2 dx
4x  x 2 4x  x 2
1 4  2x 1
  dx  4  dx
2 4x  x 2 4x  x 2
4  2x 1
Let I1   dx and I 2  dx
4x  x 2
4x  x 2
x2 1
 dx   I1 and  4I 2 ....1
4x  x 2 2
4  2x
Then, I1   dx
4x  x 2
Let 4x  x 2  t
 (4  2x)dx  dt
dt
 I1    2 t  2 4x  x 2 ... 2 
t
(Using the logarithm formula of integration,)
1
I2   dx
4x  x 2
Integrating using the power rule,
 4x  x 2    4x  x 2 
By completing square methods,
  4x  x 2  4  4 
 4  (x  2) 2
 (2) 2  (x  2) 2
1 1  x  2 
 I2   dx  sin   ... 3
(2)  (x  2)
2 2
 2 
Using equations (2) and (3) in (1), to obtain
x2 1
  1  x  2 
 4x  x 2 dx   2 2 4x  x  4sin  2   C
2

x2
  4x  x 2  4sin 1  C
 2 
x2
22.
x 2  2x  3
Ans:

Class XII Maths www.vedantu.com 57


x2 1 2(x  2)
 x  2x  3
2
dx  
2 x 2  2x  3
dx

Simplifying,
1 2x  4
  2 dx
2 x  2x  3
1 2x  2 1 2
  2 dx   2 dx
2 x  2x  3 2 x  2x  3
1 2x  2 1
  2 dx   2 dx
2 x  2x  3 x  2x  3
2x  2 1
Let I1   2 dx and I 2   2 dx
x  2x  3 x  2x  3
x2 1
 2 dx  I1  I 2 ....1
x  2x  3 2
2x  2
Then, I1   2 dx
x  2x  3
Put, x 2  2x  3  t
Integrating using the power rule,
 (2x  2)dx  dt
dt
I1    2 t  2 x 2  2x  3.. 2 
t
1
I2   2 dx.
x  2x  3
By completing square methods,
 x 2  2x  3  x 2  2x  1  2  (x  1) 2  ( 2) 2
1
 I2   dx  log (x  1)  x 2  2x  3 ... 3
(x  1)  ( 2)
2 2

Using equations (2) and (3) in (1), to obtain

x2
 dx  x 2  2x  3  log (x  1)  x 2  2x  3  C
x 2  2x  3

x3
23.
x 2  2x  5
Ans:
 x  2x  5  B
d 2
Consider (x  3)  A
dx
Class XII Maths www.vedantu.com 58
x2
 (log x) 2   x log xdx
2
We obtain the below values by equating the coefficients of x and the constant
term on both sides.
1
2A  1  A 
2
2A  B  3  B  4
1
 (x  3)  (2x  2)  4
2
1
x3 (2x  2)  4
 2 2
dx   2 dx
x  2x  5 x  2x  5
1 2x  2 1
  2 dx  4 2 dx
2 x  2x  5 x  2x  5
2x  2 1
Consider I1   2 dx and I2   2 dx
x  2x  5 x  2x  5
x 3 1
 2 dx  I1  4I2 ...1
x  2x  5 2
2x  2
Then, I1   2 dx
x  2x  5
Put x 2  2x  5  t
 (2x  2)dx  dt
Using the logarithm formula of integration,
dt
 I1    log | t | log x 2  2x  5 .... 2 
t
1
I2   2 dx
x  2x  5
1
 2 dx
 x  2x  1  6
1
 dx
(x  1) 2  ( 6) 2
1  x 1 6 
 log   .... 3
2 6  x  1  6 
We obtain the below values by substituting (2) and (3) in (1),
x 3 1 4 x 1  6
 x 2  2x  5dx  2 log x  2x  5  2 6 log x  1  6  C
2

Class XII Maths www.vedantu.com 59


1 2 x 1  6
 log x 2  2x  5  log C
2 6 x 1  6
5x  3
24.
x 2  4x  10
Ans:
5x  3  A  x 2  4x  10   B
a
dx
 5x  3  A(2x  4)  B
Equating the coefficients of x and constant term, we get
5
2A  5  A 
2
4A  B  3  B  7
5
 5x  3  (2x  4)  7
2
5
5x  3 (2x  4)  7
 2 dx   22 dx
x  4x  10 x  4x  10
5 2x  4 1
  2 dx  7  2 dx
2 x  4x  10 x  4x  10
2x  4 1
Let I1   2 dx and I 2   2 dx
x  4x  10 x  4x  10
5x  3 5
 2 dx  I1  7I 2 ....1
x  4x  10 2
2x  4
Then, I1   2 dx
x  4x  10
Consider x 2  4x  10  t  (2x  4)dx  dt
dt
 I1    2 t  2 x 2  4x  10 ...... 2 
t
1
I2   2 dx
x  4x  10
1 1
 dx   dx
 x 2
 4x  4   6 (x  2) 2
 ( 6) 2

 log (x  2) x 2  4x  10 ...  3 

Class XII Maths www.vedantu.com 60


We obtain the below values by using equations (2) and (3) in (1).
5x  3 5
 x 2  4x  10 dx  2  2 x  4x  10   7log (x  2) x  4x  10  C
2 2

 5 x 2  4x  10  7log (x  2) x 2  4x  10  C
dx
25. x2  2x  2 equals
a) xtan 1  x  1  C
b) tan1  x  1  C
c)  x  1 tan 1
xC
d) tan x  C1

Ans:
dx dx
 x 2  2x  2    x 2  2x  1  1
1
 dx
(x  1) 2  (1) 2
  tan 1 (x  1)   C
Hence, the right response is is B.

dx
26.  equals
9x  4x 2
1 1  9x  8 
a) sin  C
9  8 
1  8x  9 
b) sin 1  C
2  9 
1 1  9x  8 
c) sin  C
3  8 
1  9x  8 
d) sin 1  C
2  9 
Ans:
dx
 9x  4x 2
1
 dx
 9 
4  x 2  x 
 4 

Class XII Maths www.vedantu.com 61


By completing square methods,
1
 dx
 2 9 81 81 
4  x  x   
 4 64 64 
1
 dx
 2
9 9
2

4  x      
 8 8 
1 1
  2 2 dx
2 9  9
  x  
8  8
  9 

8    C  
x
1  dy 1 y

  a 2  y2
 sin 1    
9 
sin C
2 a 
    
  8 
Simplifying,
1  8x  9 
 sin 1  C
2  9 
Hence, the right response is B.

Exercise 7.5

x
1.
 x  1 x  2 
Ans:
x A B
Let  
(x  1)(x  2) (x  1) (x  2)
 x  A(x  2)  B(x  1)
We obtain the below values by equating the coefficients of x and the constant
term on both sides.
A  B 1
2A  B  0
On solving, we get
A  1 and B  2
x 1 2
  
(x  1)(x  2) (x  1) (x  2)
x 1 2
 dx    dx
(x  1)(x  2) (x  1) (x  2)
Class XII Maths www.vedantu.com 62
Using the logarithm formula of integration,
  log | x  1| 2log | x  2 | C
 log(x  2) 2  log | x  1| C
Simplifying,
(x  2) 2
 log C
(x  1)

1
2.
x 9
2

Ans:
1 A B
Let  
(x  3)(x  3) (x  3) (x  3)
1  A  x  3  B  x  3
Equating the coefficients of x and constant term, we get
AB0
1  3A  3B
1 1
A   and B 
6 6
1 1 1
  
(x  3)(x  3) 6(x  3) 6(x  3)
1  1 1 

 x 2  9    6(x  3) 6(x  3) 
dx   dx

Using the logarithm formula of integration,


1 1 1 | (x  3) |
  log | x  3 |  log | x  3 | C  log C
6 6 6 | (x  3) |

3x  1
3.
 x  1 x  2  x  3 
Ans:
3x  1 A B C
Let   
(x  1)(x  2)(x  3) (x  1) (x  2) (x  3)
3x  1  A(x  2)(x  3)  B(x  1)(x  3)  C(x  1)(x  2) 1
We obtain the below values by equating the coefficients of x , x 2 and the constant
term on both sides.
A BC  0
5 A  4 B  3C  3
6 A  3 B  2C  1

Class XII Maths www.vedantu.com 63


Solving these equations, to obtain
A  1, B  5, and C  4
3x  1 1 5 4
   
(x  1)(x  2)(x  3) (x  1) (x  2) (x  3)
3x  1  1 5 4 
 dx      dx
(x  1)(x  2)(x  3)  (x  1) (x  2) (x  3) 
Using the logarithm formula of integration,
 log | x  1| 5log | x  2 | 4log | x  3| C

x
4.
 x  1 x  2  x  3 
Ans:
x A B C
Let   
(x  1)(x  2)(x  3) (x  1) (x  2) (x  3)
x  A  x  2  x  3  B  x  1 x  3  C  x  1 x  2 
We obtain the below values by equating the coefficients of x , x 2 and the constant
term on both sides.
A BC  0
5 A  4 B  3 C  1
6 A 4 B 2 C 0
Solving these equations, to obtain
1 3
A  ,B  2 and C 
2 2
x 1 2 3
   
(x  1)(x  2)(x  3) 2(x  1) (x  2) 2(x  3)
x  1 2 3 
 dx      dx
(x  1)(x  2)(x  3)  2(x  1) (x  2) 2(x  3) 
Using the logarithm formula of integration,
1 3
 log | x  1| 2log | x  2 |  log | x  3 | C
2 2

2x
5.
x  3x  2
2

Ans:
2x A B
 
x  3x  2 (x  1) (x  2)
2

2 x  A  x  2   B  x  1

Class XII Maths www.vedantu.com 64


We obtain the below values by equating the coefficients of x , x 2 and the constant
term on both sides.
AB2
2 AB0
Solving these equations, we get
A  2 and B  4
2x 2 4
  
(x  1)(x  2) (x  1) (x  2)
2x  4 2 
 dx     dx
(x  1)(x  2)  (x  2) (x  1) 
Using the logarithm formula of integration,
 4log | x  2 | 2log | x  1| C

1  x2
6.
x  1  2x 
Ans:
It can be seen that the given integrand is not a proper fraction.
Therefore, on dividing 1  x 2  by x(1  2x) to obtain,
1  x2 1 1 2x 
   
x(1  2x) 2 2  x(1  2x) 
2x A B
Let   ...1
x(1  2x) x (1  2x)
 (2  x)  A(1  2x)  Bx
We obtain the below values by equating the coefficients of x , x 2 and the constant
term on both sides.
2 A  B  1 and, A  2
Solving these equations, to obtain A  2 and B  3
2x 2 3
  
x(1  2x) x 1  2x
Substituting in equation (1), we get
1  x2 1 1 2 3 
    
x(1  2) 2 2  x (1  2x) 
1  x2 1 1 2 3
 dx    (  )dx
x(1  2) 2 2 x (1  2x)
Using the power rule and logarithm formula of integration,

Class XII Maths www.vedantu.com 65


x 3 x 3
  log | x |  log |1  2x | C   log | x |  log |1  2x | C
2 2(2) 2 4

x
7.
 x 2  1  x  1
x Ax  B C
Ans: Let  2  ...1
 x  1 (x  1)  x  1 (x  1)
2

x  (Ax  B)(x  1)  C  x 2  1
x  Ax 2  Ax  Bx  B  Cx 2  C
We obtain the below values by equating the coefficients of x , x 2 and the constant
term on both sides.
AC0
A  B  1
B  C  0
1 1 1
On solving these equations, to obtain A   ,B  ,and C 
2 2 2
From equation (1), to obtain
 1 1 1
  x 
 2
x 2 2
 2  2
 x  1 (x  1) x 1 (x  1)
x 1 x 1 1 1 1
    2 dx   2 dx   dx
 x  1 (x  1) 2 x  1 2 x  1 2 x  1
2

1 2x 1 1
   2 dx  tan 1 x  log | x  1| C
4 x 1 2 2
Consider  2 dx,lel  x 2  1  t  2xdx  dt
2x
x 1
2x dt
  2 dx    log | t | log x 2  1
x 1 t
x 1 1 1
 2   log x 2  1  tan 1 x  log | x  1| C
 x  1 (x  1) 4 2 2

1 1 1
 log | x  1|  log x 2  1  tan 1 x  C
2 4 2

x
8.
 x  1  x  2 
2

Class XII Maths www.vedantu.com 66


Ans:
x
(x  1) (x  2)
2

x A B C
Let   
(x  1) 2 (x  2) (x  1) (x  1) 2 (x  2)
x  A(x  1)(x  2)  B(x  2)  C(x  1) 2
We obtain the below values by equating the coefficients of x , x 2 and the constant
term on both sides.
AC0
A  B  2 C 1
On solving, to obtain
2 2
A  and C 
9 9
1
B
3
x 2 1 2
   
(x  1) (x  2) 9(x  1) 3(x  1) 9(x∣ 2)
2 2

x 2 1 1 1 2 1
 dx   dx   dx   dx
(x  1) 2 (x  2) 9 (x  1) 3 (x  1) 2 9 (x  2)
Using the power rule and logarithm formula of integration,
2 1  1  2
 log | x  1|     log | x  2 | C
9 3  x 1 9
Simplifying,
2 x 1 1
 log  C
9 x  2 3(x  1)

3x  5
9.
x3  x2  x  1
Ans:
3x  5 3x  5

x  x  x  1 (x  1) 2 (x  1)
3 2

3x  5 A B C
let   
(x  1) (x  1) (x  1) (x  1) (x  1)
2 2

3x  5  A(x  1)(x  1)  B(x  1)  (x  1) 2


3x  5  A(x  1)2  B(x  1)  C  x 2  1  2x  (1)
We obtain the below values by equating the coefficients of x , x 2 and

Class XII Maths www.vedantu.com 67


the constant term on both sides.
AC0
B2 C 3
A  B  C  5
1 1
On solving, to obtain B  4 A   and C 
2 2
3x  5 1 4 1
   
(x  1) (x  1) 2(x  1) (x  1) 2(x  1)
2 2

3x  5 1 1 1 1 1
 dx    dx  4  dx   dx
(x  1) 2 (x  1) 2 x 1 (x  1) 2 2 (x  1)
Using the power rule and logarithm formula of integration,
1  1  1
  log | x  1| 4    log | x  1| C
2  x 1 2
1 x 1 4
 log  C
2 x  1 (x  1)

2x  3
10.
 x2  1  2x  3 
Ans:
2x  3 2x  3

 x  1 (2x  3) (x  1)(x  1)(2x  3)
2

2x  3 A B C
Let   
(x  1)(x  1)(2x  3) (x  1) (x  1) (2x  3)
 (2x  3)  A(x  1)(2x  3)  B(x  1)(2x  3)  C(x  1)(x  1)
 (2x  3)  A  2x 2  x  3  B  2x 2  5x  3  C  x 2  1
 (2x  3)  (2A  2B  C)x 2  (A  5B)x  ( 3A  3B  C)
We obtain the below values by equating the coefficients of x , x 2 and the constant
term on both sides.
2A  2B  C  0
A  5B  2
3 A  3 B  C  3
1 5 24
On solving, to obtain B   ,A  , and C  
10 2 5
2x  3 5 1 24
   
(x  1)(x  1)(2x  3) 2(x  1) 10(x  1) 5(2x  3)

Class XII Maths www.vedantu.com 68


2x  3 5 1 1 1 24 1
 dx   dx   dx   dx
 x  1 (2x  3) 2 (x  1) 10 x  1 5 (2x  3)
2

Using the logarithm formula of integration,


5 1 24
 log | x  1|  log | x  1|  log | 2x  3|
2 10 5 2
Simplifying,
5 1 12
 log | x  1|  log | x  1|  log | 2x  3 | C
2 10 5

5x
11.
 x  1  x 2  4 
Ans:
5x 5x

(x  1)  x 2  4  (x  1)(x  2)(x  2)
5x A B C
let   
(x  1)(x  2)(x  2) (x  1) (x  2) (x  2)
5 x  A  x  2  x  2   B  x  1 x  2   C  x  1 x  2 
We obtain the below values by equating the coefficients of x , x 2 and the constant
term on both sides.
A BC  0
B  3C  5 and,  4A  2B  2C  0
On solving, to obtain
5 5 5
A  ,B   , and C 
3 2 6
5x 5 5 5
   
(x  1)(x  2)(x  2) 3(x  1) 2(x  2) 6(x  2)
5x 5 1 5 1 5 1
 dx   dx   dx   dx
(x  1)  x 2  4  3 (x  1) 2 (x  2) 6 (x  2)
Using the logarithm formula of integration,
5 5 5
 log | x  1|  log | x  2 |  log | x  2 | C
3 2 6

x2  x  1
12.
x2  1
Ans:
On dividing  x 3  x  1 by x 2  1, we get

Class XII Maths www.vedantu.com 69


x3  x  1 2x  1
x 2
x 1
2
x 1
2x  1 A B
Let 2  
x  1 (x  1) (x  1)
2 x  1  A  x  1  B  x  1
We obtain the below values by equating the coefficients of x and the constant
term on both sides.
AB2
A  B  1
On solving, to obtain
1 3
A  and B 
2 2
x  x 1
3
1 3
 2 x 
x 1 2(x  1) 2(x  1)
x  x 1
3
1 1 3 1
 dx   xdx   dx   dx
x2  1 2 (x  1) 2 (x  1)
x2 3
  log | x  1|  log | x  1| C
2 2

2
13.
1  x  1  x2 
Ans:
2 A Bx  C
Let  
(1  x) 1  x 2  (1  x) 1  x 2 
2  A 1  x 2   (Bx  C)(1  x)
2  A  Ax 2  Bx  Bx 2  C  Cx
We obtain the below values by equating the coefficients of x x 2 and the constant
term on both sides.
AB0
BC 0
AC2
On solving these equations, to obtain
A  1,B  1, and C  1
2 1 x 1
  
(1  x) 1  x  1  x 1  x 2
2

2 1 x 1
 dx   dx   dx   dx
(1  x) 1  x 
2
1 x 1 x 2
1  x2

Class XII Maths www.vedantu.com 70


1 1 2x 1
  dx   dx   dx
1 x 2 1 x 2
1  x2
1
  log | x  1|  log 1  x 2  tan 1 x  C
2
3x  1
14.
 x  2
2

Ans:
3x  1 A B
Let  
(x  2) (x  2) (x  2) 2
2

 3x  1  A(x  2)  B
We obtain the below values by equating the coefficients of x and the constant
term on both sides.
A3
2A  B  1  B  7
3x  1 3 7
  
(x  2) 2 (x  2) (x  2) 2
3x  1 1 x
 dx  3 dx  7  dx
(x  2) 2
(x  2) (x  2) 2
Using the power rule and logarithm formula of integration
 1 
 3log | x  2 | 7  C
 (x  2) 
7
 3log | x  2 |  C
(x  2)

1
15.
x 1
4

Ans:
1 1 1
 2 
 x  1  x  1 x  1 (x  1)(x  1) 1  x 2 
4 2

1 A B Cx  D
Let    2
(x  1)(x  1) 1  x  (x  1) (x  1)  x  1
2

1  A(x  1) 1  x 2   B(x  1) 1  x 2   (Cx  D)  x 2  1


1  A  x 3  x  x 2  1  B  x 3  x  x 2  1  Cx 3  Dx 2  Cx  D
1  (A  B  C)x 3  (A  B  D)x 2  (A  B  C)x  (A  B  D)

Class XII Maths www.vedantu.com 71


We obtain the below values by equating the coefficients of x 3 , x 2 , x, and constant
term, we get
A BC  0
A  B  D  0
A BC  0
A  B  D  1
1 1 1
A   ,B  ,C  0, and D  
4 4 2
1 1 1 1
 4   
 x  1 4(x  1) 4(x  1) 2  x 2  1
1 1 1 1 1
 dx   log | x  1|  log | x  1|  tan x  C
x4 1 4 4 2
Simplifying,
1 x 1 1 1
 log  tan x  C
4 x 1 2

1
16. [hint: multiply numerator and denominator by x n 1 and put
x  x  1
n

xn  t ]
1
Ans:
x  x n  1
Numerator and denominator are multiplied by x n 1 , to obtain
1 x n 1 x n 1
 
x  x n  1 x n 1x  x n  1 x n  x n  1
Consider x n  t  x n 1dx  dt
1 x n 1 1 1
 dx   n n dx   dt
x  x  1
n
x  x  1 n t(t  1)
1 A B
Let  
t(t  1) t (t  1)
1  A 1  t   B t
We obtain the below values by equating the coefficients of t and constant,
A  1 and B  1
1 1 1
  
t(t  1) t (1  t)
1 1 1 1 
 dx     dx
x  x n  1 n  t (1  t) 

Class XII Maths www.vedantu.com 72


1
 [log | t |  log | t  1|]  C
n
Substitute the value of t,
1
  log x n  log x n  1   C
n
Simplifying,
1 xu
 log  C
n x 1

cos x
17. [hint: Put sinx  t ]
 1  sin x  2  sin x 
Ans:
cos x
Put,sin x  t  cos xdx  dt
(1  sin x)(2  sin x)
cos x dt
 dx  
(1  sin x)(2  sin x) (1  t)(2  t)
1 A B
let  
(1  t)(2  t) (1  t) (2  t)
1  A  2  t   B 1  t 
We obtain the below values by equating the coefficients of t and constant,
2 A  B  0 ,and 2 A  B  1
A  1 and B  1
1 1 1
  
(1  t)(2  t) (1  t) (2  t)
cos x  1 1 
 dx     dt   log |1  t |  log | 2  t | C
(1  sin x)(2  sin x) 1  t (2  t) 
Simplifying,
2t
 log C
1 t
Substitute the value of t,
2  sin x
 log C
1  sin x

18.
x 2
 1 x 2  2 
x 2
 3  x 2  4 
Ans:
Class XII Maths www.vedantu.com 73
x 2
 1 x 2  2 

 4x 2
 10 
x 2
 3 x 2  4  x 2
 3 x 2  4 

Let
 4x 2
 10 

Ax  B Cx  D

x 2
 3 x  4 
2
 x 2  3  x 2  4 
4x 2  10  (Ax  B)  x 2  4   (Cx  D)  x 2  3
4x 2  10  Ax 2  4Ax  Bx 2  4B  Cx 3  3Cx  Dx 2  3D
4x 2  10  (A  C)x 3  (B  D)x 2  (4A  3C)x  (4B  3D)
We obtain the below values by equating the coefficients of x 3 , x 2 , x and constant
term,
AC0
BD  4
4 A3 C0
4 B  3 D  10
On solving these equations, to obtain A  0.B  2.C  0,and D  6


 4x 2
 10 

2
 2
6
x 2
 3 x  4 2
 x  3  x  4 
2

x 2
 1 x 2  2 

 2

6 
 
x 2
 3 x 2  4    x 2  3  x 2  4  

 2
 x 2  1 x 2  2   2 6 
 x  3 x 2  4     x 2  3  x 2  4  
dx  1   dx

 2 6 
  1  2  2 
 x  ( 3) x  2 
2 2

 1 x   1 1 x 
 x  2 tan 1   6  tan 2   C
 3 3 2 
Simplifying,
2 x x
x tan 1  3tan 1  C
3 3 2

2x
19.
 x  1 x2  3
2

Ans:
2x
 x 2  1 x 2  3

Class XII Maths www.vedantu.com 74


Put x 2  t  2xdx  dt
2x dt
 2 dx  
 x  1 x  3
2
(t  1)(t  3)
1 A B
Let  
(t  1)(t  3) (t  1) (t  3)
I  A  t  3  B  t  1
We obtain the below values by equating the coefficients of t and
constant,
1  B  0 and 3A  B  1
On solving, we get
1 1
A  and B  
2 2
1 1 1
  
(t  1)(t  3) 2(t  1) 2(t  3)
2x  1 1 
 dx     dt
 x 2  1 x 2  3  2(t  1) 2(t  3) 
1 1
 log | (t  1) |  log | t  3 | C
2 2
Simplifying,
1 t 1 1 x2 1
 log  C  log 2 C
2 t 3 2 x 3

1
20.
x  x  1
4

Ans:
1
x  x 4  1
Numerator and denominator are multiplied by by x 3 , we get
1 x3

x  x 4  1 x 4  x 4  1
1 x3
 dx   4 4 dx
x  x 4  1 x  x  1
Consider x 4  t  4x 3dx  dt
1 1 dt
 dx  
x  x 4  1 4 t(t  1)

Class XII Maths www.vedantu.com 75


1 A B
Let  
t(t  1) t (t  1)
1  A(t  1)  Bt (1)
We obtain the below values by equating the coefficients of t and constant,
A  1 and B  1
1 1 1
  
t(t  1) t t  1
1 1  1 1 
 dx     dt
x  x 4  1 4  t t  1
Using the logarithm formula of integration,
1
 [ log | t |  log | t  1|]  C
4
Simplifying,
1 t 1 1 x4 1
 log  C  log C
4 t 4 x4

1
21. [hint: put ex  t ]
e 1
x

Ans:
1
 e  1
x

Put e x  t  e x dx  dt
1 1 dt 1
 x dx     dt
 
e  1 t  1 t t(t  1)
1 A B
Let  
t(t  1) t t  1
1  A  t  1  B t
We obtain the below values by equating the coefficients of t and constant,
A  1 and B  1
1 1 1
  
t(t  1) t t  1
1 t 1
 dt  log C
t(t  1) t
Substitute the value of t,
ex  1
 log x  C
e

Class XII Maths www.vedantu.com 76


xdx
22.   x  1 x  2  equals

 x  1
2

a) log C
x2

 x  2
2

b) log C
x1

 x1
2

c) log   C
 x2
d) log  x  1 x  2   C
Ans:
x A B
Let  
(x  1)(x  2) (x  1) (x  2)
x  A  x  2   B  x  1
We obtain the below values by equating the coefficients of x and constant,
A  1 and B  2
x 1 2
  
(x  1)(x  2) (x1) (x  2)
x  1 2 
 dx     dx
(x  1)(x  2)  (x  1) (x  2) 
Using the logarithm formula of integration,
  log | x  1| 2log | x  2 | C
Simplifying,
(x  2) 2
 log C
x 1
Thus, the right response is B.

dx
23.  x x
2
 1
equals

a. log x  log  x 2  1  C
1
2
b. log x  log  x 2  1  C
1
2

Class XII Maths www.vedantu.com 77


c.  log x  log  x 2  1  C
1
2
log x  log  x 2  1  C
1
d.
2
Ans:
1 A Bx  C
Let  ,
x  x 2  1 x x 2  1
1  A  x 2  1  (Bx  C)x
We obtain the below values by equating the coefficients of x 2 , x, and constant
term,
A  B  0, C  0 A 1
On solving these equations, to obtain
A  1,B  1, and C  0
1 1 x
   2
x  x ,1 x x  1
2

1 1 x 
 dx     2 dx
x  x  1
2
 x x  1
1
 log | x |  log x 2  1  C
2
Thus, the right response is A.

Exercise 7.6

1. xsinx
Ans:
Let I   xsin xdx
Consider u  x and v  sin x and integrating by parts, to obtain
 d  
I   x sin xdx    x   sinxdx dx
 dx  
 x( cos x)  1.( cos x)dx
 xcos x  sin x  C

2. xsin3x
Ans:
Let I   xsin3xdx
Consider u  x and v  sin3x and integrating by parts, to obtain

Class XII Maths www.vedantu.com 78


 d  
I  x  sin3xdx    x   sin3xdx 
 dx  
  cos3x    cos3x 
 x   1    dx
 3   3 
 x cos3x 1  x cos3x 1
   cos3xdx   sin 3x  C
3 3 3 9

3. x 2ex
Ans:
Let I   x 2e x dx
Consider u  x 2 and v  e x
 d  
I  x 2  e x dx    x 2   e x dx dx
 dx  
 x e   2x  e dx
2 x x

 x 2ex  2 x  ex dx
Again using integration by parts, to obtain
  d   
 x 2e x  2  x   e x dx    x 2   e x dx dx 
  dx   
 x 2e x  2  xe x   e x dx 
Simplifying,
 x 2ex  2  xe x  e x 
 x 2e x  2xe x  2e x  C
 ex  x 2  2x  2   C

4. xlog x
Ans:
Let I   x log xdx
Consider u  log x and v  x and integrating by parts, to obtain
 d  
I  log x  xdx    log x   xdx dx
 dx  
2 2
x 1 x
 log x     dx
2 x 2
2
x log x x x 2 log x x 2
  dx   C
2 2 2 4
Class XII Maths www.vedantu.com 79
5. xlog 2x
Ans:
Let I   x log 2xdx
Consider u  log 2x and v  x and integrating by parts, to obtain
 d  
I  log 2x  xdx    2log x   xdx dx
 dx  
2 2
x 2 x
 log 2x     dx
2 2x 2
2
x log 2x x
   dx
2 2
Integrating using the power rule
x 2 log 2x x 2
  C
2 4

6. x 2 log x
Ans:
Let I   x 2 log xdx
Consider u  log x and v  x 2 and integrating by parts, to obtain
 d  
I  log x  x 2dx    log x   x 2dx dx
 dx  
x 
3
1 x 3

 log x      dx
 3 x 3
Integrating using the power rule
x 3 log x x2 x 3 log x x 3
   dx   C
3 3 3 9

7. xsin 1 x
Ans:
Let I   xsin 1 xdx
Consider u  sin 1 x and v  x and integrating by parts, to obtain
 d  
I  sin 1 x  xdx   sin 1 x   xdx dx
 dx  
x 2
1 x 2

 sin 1 x      dx
 2 1 x 2
2

Class XII Maths www.vedantu.com 80


x 2 sin 1 x 1  x 2
   dx
2 2 1  x2
Adding and subtracting by 1
x 2 sin 1 x 1  1  x 2 1 
    dx
2 2  1  x2 1  x2 
Simplifying,
x 2 sin 1 x 1  1 
    1  x2  dx
2 2  1 x 
2

x sin x 1 
2 1
1 
    1  x 2 dx   dx 
2 2 1 x 2

1
x sin x 1  x
2
1 
   1  x 2  sin 1 x  sin 1 x   C
2 2 2 2 
Simplifying,
x 2 sin 1 x x
1  x 2  sin 1 x  sin 1 x  C   2x 2  1 in 1 x 
1 1 1 x
  1 x2
2 4 4 2 4 4
C

8. xtan 1 x
Ans:
Let I   x tan 1 xdx
Consider u  tan 1 x and v  x and integrating by parts, to obtain
 d  
I  tan 1 x  xdx    tan 1 x   xdx dx
 dx  
 x2  1 x2 x 2 tan 1 x 1 x 2
 tan 1 x     dx    dx
 
2 1  x 2
2 2 2 1  x 2

Adding and subtracting by -1

x 2 tan 1 x 1  x 2  1 1  x 2 tan 1 x 1  1 
        dx
2  1  x 2 1  x 2 
dx 1
2 2 2  1  x2 
Simplifying,
x 2 lan 1 x 1 x2
  x  tan x   C  tan 1 x   tan 1 x  C
x 1
 1

2 2 2 2 2

9. xcos 1 x
Ans:

Class XII Maths www.vedantu.com 81


Let I   x cos 1 xdx
Taking u  cos 1 x and v  x and integrating by parts, to obtain
 d  
I  cos 1 x  xdx    cos 1 x   xdx dx
 dx  
x 2
1 x 2

 cos 1 x    dx
2 1 x 2
2
Adding and subtracting by -1
x 2 cos 1 x 1 1  x 2  1
   dx
2 2 1  x2
Simplifying,

x 2 cos 1 x 1   1  
    1  x2   2 
dx
2 2   1  x 
x 2 cos 1 x 1 1  1 
   1  x 2 dx   
2  1  x 2 
dx
2 2
x 2 cos 1 x 1 1
  I1  cos 1 x....1
2 2 2
Where I1   1  x dx
2

d x 2
 I1  x  1  x   2
1  x  xdx  I1  x 1  x  
2 2
dx
dx 1  x2
1  x2 1  dx 
 I1  x 1  x 2   dx  I  x 1  x 2
   1  x 2
dx   
1  x2 1  x2 
1

 I1  x 1  x 2  I1  cos 1 x  2I1  x 1  x 2  cos 1 x
x 1
 I1  1  x 2  cos 1 x
2 2
Substituting in (1), we get
x cos 1 x 1  x 1  1
I   1  x 2  cos 1 x   cos 1 x
2 2 2 2  2
Simplifying,


 2x 2  1
cos 1 x 
x
1  x2  C
4 4

10.  sin 1 x 
2

Ans:

Class XII Maths www.vedantu.com 82


Let I    sin 1 x   1dx
2

Consider u   sin 1 x  and v  1 and integrating by parts, to obtain


2

d 
I    sin 1 x   1dx     sin 1 x   1.dx dx
2

 dx 
1

  sin 1 x  x  
2 2sin x
 xdx
1  x2
 2x 
 x  sin 1 x    sin 1x  
2

2 
dx
 1  x 
 2x  d  2x  
 x  sin 1 x   sin 1 x 
2
   dx 
1
dx  sin x dx dx 
 1  x2   1  x2  
 
 x  sin 1 x   sin 1 x  2 1  x 2  
2 1
 2 1  x 2
dx 
 1  x2 
 x  sin x   2 1  x sin x   2dx
1 2 2 1

 x  sin 1 x   2 1  x 2 sin 1 x  2x  C
2

xcos 1 x
11.
1  x2
Ans:
x cos 1 x
Let I   dx
1 x 2

Multiplying and dividing by 2


1 2x
I   cos 1
xdx
2 1  x2
 2x 
Consider u  cos 1 x and v   2 
and integrating by parts, to obtain
 1  x 
1  2x  d  2x  
I  cos 1 x  dx     cos 1
x   dx dx 
2  1  x2  dx  1  x2  
1  1  1 
 cos 1 x  2 1  x 2    2 1  x 2
dx   2 1  x 2
cos 1
x   2dx 
2  1 x 2
 2  
Simplifying,
1
  2 1  x 2 cos 1 x  2x   C
2  

Class XII Maths www.vedantu.com 83


   1  x 2 cos 1 x  x   C
 

12. xsec2 x
Ans:
Let I   xsec2 xdx
Consider u  x and v  sec2 x and integrating by parts, to obtain
 d  
I  x  sec2 xdx    x   sec 2 xdx dx
 dx  
 x tan x  1  tan xdx
 x tan x  log | cos x | C

13. tan 1 x
Ans:
Let I  1  tan 1 xdx
Consider u  tan 1 x and v  1 and integrating by parts, to obtain
 d   1
I  tan 1 x 1dx    tan 1 x  1.dx dx  tan 1 xx   xd
 dx   1  x 2

1 2x
 x tan 1 x   dx
2 1  x2
1
 x tan 1 x  log 1  x 2  C
2
 x tan 1 x  log 1  x 2   C
1
2

14. x  log x  dx
2

Ans:
I   x(log x) 2 dx
Consider u  (log x) 2 and v  1 and integrating by parts, to obtain
  d  
2

I  (log)  xdx     log x    xdx dx
2

  dx   
x2  1 x2 
 (log x)    2log x   dx 
2

2  x 2 
2
x
 (log x) 2   x log xdx
2

Class XII Maths www.vedantu.com 84


Again using integration by parts, to obtain
x2   d   
I  (log x) 2  log x  xdx    log x   xdx dx 
2   dx   
x 2
x 2
1 x 2

 (log x) 2    log x    dx 
2 2 x 2 
2 2
x x 1 x2 x2 x2
 (log x)  log x   xdx  (log x)  log x   C
2 2

2 2 2 2 2 4

15.  x2  1 log x
Ans:
Let I    x 2  1 log xdx   x 2 log xdx   logxdx
Let I  I1  I2 (1)
Where, I1   x 2 log xdx and I2   logxdx
I1   x 2 log xdx
Consider u  log x and v  x 2 and integrating by parts, to obtain
 d  
I1  log x   x 2dx    log x   x 2dx dx
 dx  
3 3 3
x 1 x x 1
 log x     dx  log x   x 2dx
3 x 3 3 3
3 3
x x
 log x   C1 (2)
3 9
I2   logxdx
Consider u  log x and v  1 and integrating by parts, to obtain
 d  
I2  log x 1.dx    log x  1.dx 
 dx  
1
 log x  x   xdx
x
 x log x  x.. 3
Using equations (2) and (3) in (1), we get
x3 x3
I  log x   C1  x log x  x  C2
3 9
3 3
x x
 log x   x log x  x   C1  C2 
3 9

Class XII Maths www.vedantu.com 85


 x3  x3
   x  log x   x  C
 3  9

16. ex  sin x  cos x 


Ans:
Consider I   ex (sin x  cos x)dx
Consider f (x)  sin x
f  (x)  cos x
I   ex f (x)  f  (x) dx
Since,  ex f (x)  f  (x) dx  e xf (x)  C
 I  e x sin x  C

xe x
17.
1  x 
2

Ans:
xe x x  x 
Consider I   dx   (1  x) 2  dx
e 
(1  x) 2  
1  x  1   1 1 
  ex  2 
dx   e x   2 
dx
 (1  x)   1  x (1  x) 
1 1
Here, f (x)  f  (x) 
1 x (1  x) 2
xe x
 dx   e x f (x)  f  (x) dx
(1  x) 2

Since,  ex f (x)  f  (x) dx  e xf (x)  C


xe x ex
 dx  C
(1  x) 2 1 x

 1  sin x 
18. Integrate the function - e x  
 1  cos x 
 1  sin x 
Ans: First simplify – e x  
 1  cos x 
It is known that –
x x x x
1  sin x  sin 2  cos 2  2sin cos
2 2 2 2
Class XII Maths www.vedantu.com 86
x
1  cos x  2cos 2
2
 2x 2 x x x
 sin  cos  2sin cos
 1  sin x  x
 ex   2 2 2 2
 e  
 1  cos x   2cos 2
x

 2 
 x x 
2

  sin  cos  
 ex  
2 2 
 2cos 2
x 
 2 
 
 x x 
2

 sin  cos  
1 x   2 2 
 e
2  cos 2
x 
 2 
 
2
 x x
 sin  cos
1
 ex  2 2
x 
2  cos 
 2 
2
 x x
1 x  sin 2 cos 2 
 e   
2  cos x cos x 
 2 2
2
1  x 
 e x  tan  1
2  2 
1  x x
 e x  tan 2  1  2 tan 
2  2 2
x x
But, 1  tan 2  sec 2
2 2
1  x x
 e x  sec 2  2 tan 
2  2 2
1 x x
 e x  sec 2  tan 
2 2 2
 1  sin x  x  1 2 x x
 ex    e  sec  tan 
 1  cos x  2 2 2

Class XII Maths www.vedantu.com 87


It is known that,  ex f (x)  f '(x) dx e xf (x)  C
x 1 x
If we say, f (x)  tan  f '(x)  sec 2
2 2 2
 1  sin x  x
Thus, we get –  e x  dx  e tan  C
x

 1  cos x  2

1 1 
19. Integrate the function - ex   2 
x x 
1 1 
Ans: Say, I   e x   2  dx
x x 
1 1
Suppose, f (x)   f '(x)   2
x x
It is known that,  e f (x)  f '(x) dx e xf (x)  C
x

1 1  ex
Thus, we get – I   e   2  dx   C
x

x x  x

(x  3)e x
20. Integrate the function -
(x  1)3
(x  3) x  (x  1  2) 
Ans:  e x dx    (x  1)3  dx
e
(x  1)3  
 (x  1) 2 
  ex   3 
dx
 (x  1) 3
(x  1) 
 1 2 
  ex   3 
dx
 (x  1) (x  1) 
2

1 2
Suppose, f (x)   f '(x)  
(x  1) 2
(x  1)3
It is known that,  ex f (x)  f '(x) dx e xf (x)  C
(x  3) ex
Thus,  e x
dx  C
(x  1)3 (x  1) 2

21. Integrate the function - e2x sin x


Ans: Say, I   e2x sin xdx
Perform Integration by parts –  uvdx  u  vdx    u '  vdx  dx

Class XII Maths www.vedantu.com 88


With – u  sin x v  e2x
 d  
I   e2x sin xdx  sin x  e2x dx    sin x   e2x dx  dx
 dx  
e 2x
 e 
2x

 sin x    cos x   dx
2  2 
2x

   e2x cos x  dx
e 1
 sin x
2 2
Perform Integration by parts for –   e2x cos x  dx
e2 x 1   d   
 sin x  cos x  e 2 x dx    cos x   e 2 x dx  dx 
2 2  dx   
e 2x
1 e 2x
 e  
2x

 sin x  cos x     sin x   dx 


2 2 2  2  
e2x 1  e2x 1 
 sin x  cos x   (sin x)e2x dx 
2 2 2 2 
  (sin x)e2x dx
2x 2x
e e cos x 1
 sin x 
2 4 4
But, I   e sin xdx
2x

e2x e2x cos x 1


 I  sin x   I
2 4 4
2x 2x
1 e e cos x
 I  I  sin x 
4 2 4
2x 2x
5 e sin x e cos x
 I 
4 2 4
2x 2x
5 2e sin x e cos x
 I 
4 4 4
 5I  e (2sin x  cos x)
2x

e2x
Thus, we get – I  (2sin x  cos x)  C
5

 2x 
22. Integrate the function - sin 1  3 
1 x 
Ans: Say, x  tan   dx=sec2d
 2x  1  2 tan  
 sin 1  3 
 sin  
1 x   1  tan  
3

Class XII Maths www.vedantu.com 89


2 tan 
But, sin 2 
1  tan 3 
 2x   2 tan   1  2x 
 sin 1  3 
 sin 1    sin  3 
 sin 1  sin 2   2
1 x   1  tan   1 x 
3

 2x 
Therefore,  sin 1  3 
dx   2sec 2d
1 x 
 2 sec d
2

Perform Integration by parts –  uvdx  u  vdx    u '  vdx  dx


With – u   v  sec2 
  d   
2 sec 2d  2  sec 2 d       sec 2 d  d 
  d   
 2 tan     tan  d
 2 tan   ( log | cos  |)  C
 2 tan   log | cos  |  C
Replace   tan 1 x
 2tan 1 x tan(tan 1 x)  log | cos(tan 1 x) |  C
1
It is known that – tan 1 x  cos 1
1  x2
 1 
 2 tan 1 x(x)  log | cos(cos 1 ) | C
 1  x2 
 1 
 2  x tan 1 x  log | | C
 1  x2 
  
1

 2  x tan x  log(1  x ) 2   C
1 2

 
Here, log m  n log m
n

 1 
 2  x tan 1 x  log(1  x 2 )   C
 2 
 2x tan x  log(1  x )  C
1 2

 2x 
Thus,  sin 1  3 
dx  2x tan 1 x  log(1  x 2 )  C
1 x 

23. Choose the correct answer:  x 2e x dx equals


3

Class XII Maths www.vedantu.com 90


1 x
e C
3
(a)
3
1
(b) e x  C
2

3
1
(c) e x  C
3

2
1
(d) e x  C
2

2
Ans: Say, I   x 2e x dx
3

Suppose, t  x 3  dt  3x 2dx
1 t
Rewriting the equation – I   x 2e x dx   e dt
3

3
1 t 1
I
3
 e dt  e t  C
3
Replacing t  x 3

1
 I  ex  C
3

3
The correct option is A.

24. Choose the correct answer:  ex secx(1  tan x)dx


(a) ex cos x  C
(b) ex secx  C
(c) ex sin x  C
(d) ex tan x  C
Ans: Say, I   ex sec x(1  tan x)dx
 I   ex (sec x  sec x tan x)dx
Suppose, f (x)  sec x  f '(x)  sec x tan x
It is known that,  ex f (x)  f '(x) dx e xf (x)  C
 I   ex (sec x  sec x tan x)dx  e x sec x  C
Thus, I  e x sec x  C
The correct option is B.

Exercise 7.7

1. Integrate the function - 4  x2

Class XII Maths www.vedantu.com 91


Ans: Say, I   4  x 2 dx   22  x 2 dx
x 2 a 2 1 x
It is known that –  a  x dx 2 2
a  x  sin
2
C
2 2 a
x 2 22 x
 I   22  x 2 dx  2  x 2  sin 1  C
2 2 2
x x
I 4  x 2  2sin 1  C
2 2
x x
Thus,  4  x 2 dx  4  x 2  2sin 1  C
2 2

2. Integrate the function - 1  4x 2


Ans: Say, I   1  4x 2 dx   12  (2x) 2 dx
Let, 2x  t  2dx  dt
t dt
x   dx 
2 2
2
 t  dt 1
So, we get – I   1   2( )    12   t  dt
2 2

 2  2 2
1
I  12   t  dt
2

2
x 2 a2 x
It is known that –  a 2  x 2 dx 
2
a  x 2  sin 1  C
2 a
1t 1 
 I   1  t 2  sin 1 t   C
2 2 2 
t 1 
 I   1  t 2  sin 1 t   C
4 4 
Replace – t  2x
 2x 1 
I 1  (2x) 2  sin 1 2x   C
 4 4 
x 1 
 I   1  4x 2  sin 1 2x   C
2 4 

x 1
Thus,  1  4x 2 dx 
2
1  4x 2  sin 1 2x  C
4

3. Integrate the function - x 2  4x  6

Class XII Maths www.vedantu.com 92


Ans: First simplify – x 2  4x  6
x 2  4x  6  x 2  4x  4  2
 (x 2  4x  4)  2  (x  2) 2  ( 2) 2
 x 2  4x  6  (x  2) 2  ( 2) 2
  x 2  4x  6dx   (x  2) 2  ( 2) 2 dx
x 2 a2
It is known that –  x 2  a 2 dx 
2
x  a 2  log x  x 2  a 2  C
2
x2
  (x  2) 2  ( 2) 2 dx  (x  2) 2  ( 2) 2
2
( 2) 2
 log (x  2)  (x  2) 2  ( 2) 2  C
2
x2 2 2
 x  4x  6  log (x  2)  x 2  4x  6  C
2 2
x2 2
Thus,  x 2  4x  6dx  x  4x  6  log (x  2)  x 2  4x  6  C
2

4. Integrate the function - x 2  4x  1


Ans: First simplify – x 2  4x  1
x 2  4x  1  x 2  4x  4  3
 (x 2  4x  4)  3  (x  2) 2  ( 3) 2
 x 2  4x  1  (x  2) 2  ( 3) 2
  x 2  4x  1dx   (x  2) 2  ( 3) 2 dx
x 2 a2
It is known that –  x 2  a 2 dx 
2
x  a 2  log x  x 2  a 2  C
2
x2
  (x  2) 2  ( 3) 2 dx  (x  2) 2  ( 3) 2
2
( 3) 2
 log (x  2)  (x  2) 2  ( 3) 2  C
2
x2 2 3
 x  4x  1  log (x  2)  x 2  4x  1  C
2 2
x2 2 3
Thus,  x 2  4x  1dx  x  4x  1  log (x  2)  x 2  4x  1  C
2 2

5. Integrate the function - 1  4x  x 2

Class XII Maths www.vedantu.com 93


Ans: First simplify –1  4x  x 2
1  4x  x 2  1  4x  x 2  4  4  1  4  (x 2  4x  4)
 5  (x 2  4x  4)  ( 5) 2  (x  2) 2
 1  4x  x 2  ( 5) 2  (x  2) 2
  1  4x  x 2 dx   ( 5) 2  (x  2) 2 dx
x 2 a 2 1 x
It is known that –  a  x dx 
2 2
a  x  sin
2
C
2 2 a
x2 ( 5) 2 1 x  2
  ( 5)  (x  2) dx 
2 2
( 5)  (x  2) 
2 2
sin C
2 2 5
x2 5 x2
 1  4x  x 2  sin 1 C
2 2 5
x2 5 x2
Thus,  1  4x  x 2 dx  1  4x  x 2  sin 1 C
2 2 5

6. Integrate the function - x 2  4x  5


Ans: First simplify – x 2  4x  5
x 2  4x  5  x 2  4x  5  4  4  (x 2  4x  4)  5  4
 (x 2  4x  4)  9  (x  2) 2  (3) 2
 x 2  4x  5  (x  2) 2  (3) 2
  x 2  4x  5dx   (x  2) 2  (3) 2 dx
x 2 a2
It is known that –  x  a dx 
2 2
x  a  log x  x 2  a 2  C
2

2 2
x2 (3) 2
  (x  2)  (3) dx 
2 2
(x  2)  (3) 
2 2
log (x  2)  (x  2) 2  (3) 2  C
2 2
x2 2 9
 x  4x  5  log (x  2)  x 2  4x  5  C
2 2
x2 2 9
Thus,  x 2  4x  5dx  x  4x  5  log (x  2)  x 2  4x  5  C
2 2

7. Integrate the function - 1  3x  x 2


Ans: First simplify –1  3x  x 2
9 9 9 9
1  3x  x 2  1  x 2  3x    1   (x 2  3x  )
4 4 4 4

Class XII Maths www.vedantu.com 94


2
94 9  13  
2
9  13  3
  (x  3x  )     (x  3x  )  
2 2
 x  
4 4 4 4  2   2
2
 13   3
2

 1  3x  x   2
 x  
 2   2
2
 13   3
2

  1  3x  x dx   
2
   x   dx
 2   2
x 2 a2 x
It is known that –  a 2  x 2 dx  a  x 2  sin 1  C
2 2 a
 3
2
 x  2
 13   3
2
 2   13   3
2

     x   dx    x  
 2   2  2  2   2
2
 13   3
   x 
2 
 sin 1 
2
C
2  13 
 
 2 
2x  3 13 2x  3
 1  3x  x 2  sin 1 C
4 8 13
2x  3 13 2x  3
Thus,  1  3x  x 2 dx  1  3x  x 2  sin 1 C
4 8 13

8. Integrate the function - x 2  3x


Ans: First simplify – x 2  3x
9 9 9 9
x 2  x  x 2  3x    (x 2  3x  ) 
4 4 4 4
2 2 2
 3 9  3 3
x     x    
 2 4  2 2
2 2
 3 3
 x  3x   x     
2

 2 2
2 2
 3 3
  x  3xdx  
2
 x      dx
 2 2

Class XII Maths www.vedantu.com 95


x a2
It is known that –  x  a dx 
2 2
x  a  log x  x 2  a 2  C
2 2

2 2
 3
2 2  x  2 2
 3 3  3 3
   x      dx  
2
 x    
 2 2 2  2 2
2
3
  2 2
 2  3  3 3
 log  x     x       C
2  2  2 2
2x  3 2 9  3
 x  3x  log  x    x 2  3x  C
4 8  2
2x  3 2 9  3
Thus,  x 2  3xdx 
4
x  3x  log  x    x 2  3x  C
8  2

x2
9. Integrate the function - 1 
9
2
x
Ans: First simplify –1 
9
2
x 1 1
1  (9  x 2 )  (32  x 2 )
9 9 9
x2 1 2 1
 1  (3  x 2 )  (32  x 2 )
9 9 3
x2 1 1
  1  dx   (32  x 2 )dx   (32  x 2 )dx
9 3 3
x 2 a2
It is known that –  x 2  a 2 dx  x  a 2  log x  x 2  a 2  C
2 2
1 1 x (3) 2

  (32  x 2 )dx   (x) 2  (3) 2  log (x)  (x) 2  (3) 2   C
3 32 2 
1 x 2 9 
  x  9  log x  x 2  9   C
32 2 
x 2 3
 x  9  log x  x 2  9  C
6 2
2
x x 2 3
Thus,  1  dx  x  9  log x  x 2  9  C
9 6 2

Class XII Maths www.vedantu.com 96


10. Choose the correct answer:  1  x 2 dx is equal to –
x 1
(a) 1  x 2  log (x  1  x 2 )  C
2 2
3
2
(b) (1  x 2 ) 2  C
3
3
2
(c) x(1  x )  C
2 2

3
x2 1
(d) 1  x 2  x 2 log (x  1  x 2 )  C
2 2
x 2 a2
Ans: It is known that –  x  a dx 
2 2
x  a  log x  x 2  a 2  C
2

2 2
2
x 2 2 1
Thus,  x 2  12 dx  x  1  log x  x 2  12  C
2 2
x 2 1
 x  1dx  2 x  1  2 log x  x  1  C
2 2

The correct answer is option A.

11. Choose the correct answer:  x 2  8x  7dx is equal to –


1
(a) (x  4) x 2  8x  7  9log (x  4  x 2  8x  7 )  C
2
1
(b) (x  4) x 2  8x  7  9log (x  4  x 2  8x  7 )  C
2
1
(c) (x  4) x 2  8x  7  3 2 log (x  4  x 2  8x  7)  C
2
1 9
(d) (x  4) x 2  8x  7  log (x  4  x 2  8x  7 )  C
2 2
Ans: First simplify – x  8x  7
2

x 2  8x  7  9  9  x 2  8x  16  9  (x 2  8x  16)  9
 (x  4) 2  (3) 2
 x 2  8x  7  (x  4) 2  (3) 2
  x 2  8x  7dx   (x  4) 2  (3) 2 dx
x 2 a2
It is known that –  x  a dx  x  a  log x  x 2  a 2  C
2 2 2

2 2

Class XII Maths www.vedantu.com 97


(x  4) (3) 2
  (x  4)  (3) dx 
2 2
(x  4)  (3) 
2 2
log (x  4)  (x  4) 2  (3) 2  C
2 2
x4 2 9
 x  8x  7  log (x  4)  x 2  8x  7  C
2 2
x 4 2 9
Thus,  x 2  8x  7dx  x  8x  7  log (x  4)  x 2  8x  7  C
2 2
The correct answer is option D

Exercise 7.8
b

1. Evaluate the definite integral as limit of sum –  xdx


a

Ans: The definite integral as a limit of sum is given by –


b
1
a f (x)dx  (b  a)lim
n 
f (a)  f (a  h)  ...  f (a  (n  1)h 
n
ba
With, h   0 as n  
n
ba
Here, f (x)  x ; a  a ; b  b ; h 
n
b
1
  xdx  (b  a)lim
n 
a  (a  h)  (a  2h)  ...  (a  (n  1)h)
a n
1
 (b  a)lim (a  a  a  ...  a) n  times  (h  2h  ...  (n  1)h) 
n 
n
1
 (b  a)lim  na  h(1  2  3  ...  (n  1)) 
n 
n
n(n  1)
It is known that the sum of n-terms is – 1  2  3  ...  (n  1) 
2
1  n(n  1)  
 (b  a)lim na  h  
n 
n  2 
‘n’ is a common factor
1   h(n  1)  
 (b  a)lim .n a   
n 
n   2  
  h(n  1)  
 (b  a)lim a   
n 
  2 
Replace the value for ‘h’

Class XII Maths www.vedantu.com 98


 ba  
   n  (n  1)  
 (b  a)lim a     
n 
  2 
  

  (b  a)(n  1) 
 (b  a)lim a   
n 
  2n 
   n  1  
  (b  a)  n   
 (b  a)lim a     
n 
  2 
  

   1  
  (b  a) 1  n   
 (b  a)lim a     
n 
  2 
   

1
Here, as n     0
n
  (b  a) 1  0   
 (b  a) a   
  2 
  b  a 
 (b  a) a   
  2 
 2a  b  a  
 (b  a)  
 2 
ab
 (b  a)  
 2 
 b2  a 2 
 
 2 
b
1
Thus,  xdx  (b2  a 2 )
a 2
5

2. Evaluate the definite integral as limit of sum –  (x  1)dx


0

Ans: The definite integral as a limit of sum is given by –


b
1
a f (x)dx  (b  a)lim
n 
f (a)  f (a  h)  ...  f (a  (n  1)h 
n

Class XII Maths www.vedantu.com 99


ba
With, h   0 as n  
n
ba 50 5
Here, f (x)  x  1 ; a  0 ; b  5 ; h   
n n n
5
1  5   10   (n  1)5  
  (x  1)dx  (5  0)lim f (0)  f    f    ...  f  
0
n 
n n  n   n 
1   5   10   (n  1)5  
 5lim 1    1    1  ...    1 
n 
n n   n   n 
1  5   (2)5   (n  1)5  
 5lim (1  1  1...  1) n  times        ...   
n 
n n  n   n 
1 5 
 5lim  n  (1  2  3  ...  (n  1) 
n 
n n 
n(n  1)
It is known that the sum of n-terms is – 1  2  3  ...  (n  1) 
2
1 5  n(n  1)  
 5lim  n   
n 
n n  2  
‘n’ is a common factor
1   5(n  1)  
 (b  a)lim .n 1   
n 
n   2n  
  5(n  1)  
 5lim 1   
n 
  2n  
   n  1  
  5 n  
 5lim 1     
n 
  2 

  

   1  
  5 1  n   
 5lim 1     
n 
  2 
  

1
Here, as n     0
n
  5 1  0   
 5 1   
  2 

Class XII Maths www.vedantu.com 100


  5 
 5 1    
  2 
7
 5 
2
 35 
 
 2 
5
35
Thus,  (x  1)dx 
0 2
3

3. Evaluate the definite integral as limit of sum –  x 2dx


2

Ans: The definite integral as a limit of sum is given by –


b
1
a f (x)dx  (b  a)lim n 
f (a)  f (a  h)  ...  f (a  (n  1)h 
n
ba
With, h   0 as n  
n
32 1
Here, f (x)  x 2 ; a  2 ; b  3 ; h  
n n
3
1  1  2  (n  1)  
  x 2dx  (3  2)lim f (2)  f  2    f  2    ...  f  2  
2
n 
n  n  n  n  
1 2  (n  1)  
2 2 2
1  2 
 1lim  2   2     2    ...   2   
n 
n  n  n  n  
 2  2 1 1   2 22 2 
 2   2   2.2.   2  2  2.2.   ...
 n  n
2
1 n n 
 lim
n 
n   2 (n  1) 2 (n  1)  
  2   2.2.  
 
2
 n n 
 2 1 2 2
(n  1) 2

2  2  ...2 n  times   2  2  ...  


2 2

1 n n n 2 
 lim
n 
n  1 2 (n  1)  
  2.2.  2.2.  ...  2.2.  
  n n n  
1 2
  1  2  ...  (n  1)
1 2 1
 lim  2 n  1  2 2
 ...  (n  1) 2
 2.2.
n 
n n 2
n 
1 
 
1 2 4
 lim 4n  1  2 2
 ...  (n  1) 2
 1  2  ...  (n  1) 
n 
n  n 2
n 

Class XII Maths www.vedantu.com 101


n(n  1)
It is known that the sum of n-terms is – 1  2  3  ...  (n  1) 
2
It is also known that the sum of n-squared-terms is –
n(n  1)(2n  1)
12  22  32  ...  (n  1) 2 
6
1 1  n(n  1)(2n  1)  4  n(n  1) 
 lim  4n  2    
n 
n n  6  n  2 
1 1  (n  1)(2n  1)   (n  1) 
 lim  4n     4 
n 
n n 6   2 
1
Taking inside
n
 1  (n  1)(2n  1)  4  (n  1) 
 lim  4  2    
n 
 n  6  n  2 
   (n  1)  (2n  1)   
   n  n    (n  1)  
   
 lim 4   2  
n 
  6   n  
   
   1  1  
   n     1  
1 2
4     n  
 lim   2 1   
n 
  6   n  
   
1
Here, as n     0
n
  1  0  2  0   
 4     21  0
  6  
  1 2   
 4     21
  6  
 1  
 4     2
 3 
12  1  6

3
19

3

Class XII Maths www.vedantu.com 102


3
19
Thus,  x 2dx 
2 3
4

4. Evaluate the definite integral as limit of sum –  (x2  x)dx


1

Ans: The definite integral as a limit of sum is given by –


b
1
a f (x)dx  (b  a)lim
n 
f (a)  f (a  h)  ...  f (a  (n  1)h 
n
ba
With, h   0 as n  
n
Let us say I  I1  I2
4 4
With, I1   x dx ; I2   xdx
2

1 1
4
Finding I1   x 2dx
1

4 1 3
Here, f (x)  x 2 ; a  1 ; b  4 ; h  
n n
4
1  3   (2)3   (n  1)3  
 I1   x 2dx  (4  1)lim f (1)  f 1    f 1    ...  f 1  
1
n 
n  n  n   n  
1 2  3  6  3(n  1)  
2 2 2

 3lim 1  1    1    ...  1   
n 
n  n  n  n  
 2  2 32 3   2 22.32 2.3  
 1  1   2.2.   1   2.2.   ...
1  n2 n  n2 n  
 3lim
n  
n  2 3 (n  1) 2 2
(n  1)3  
  1   2.2.  
  n2 n  
 2 2  32 22.32 (n  1) 2 .32 
   n times  n 2 n 2
    
2
1 1 ...1 ...
1  n2 
 3lim
n 
n  3 2.3 (n  1).3  
  2.  2.  ...  2.  
  n n n  
1 2

 3lim 12 n  2 12  22  ...  (n  1) 2   2. 1  2  ...  (n  1)
3 3
n 
n n n 
1 
 
9 2 6
 3lim n  1  2 2
 ...  (n  1) 2
  1  2  ...  (n  1) 
n 
n  n 2
n 

Class XII Maths www.vedantu.com 103


n(n  1)
It is known that the sum of n-terms is – 1  2  3  ...  (n  1) 
2
It is also known that the sum of n-squared-terms is –
n(n  1)(2n  1)
12  22  32  ...  (n  1) 2 
6
1 9  n(n  1)(2n  1)  6  n(n  1) 
 3lim  n  2    
n 
n n  6  n  2 
1 3  (n  1)(2n  1)  
 3lim n     3(n  1) 
n 
n n 2  
1
Taking inside
n
 3  (n  1)(2n  1)  3 
 3lim 1  2    (n  1) 
n 
 n  2  n 
   (n  1)  (2n  1)   
   n  n    (n  1)  
 3lim 1  3     3
  
n 
  2   n 
   
   1  1  
     
n    1  
 1 2
 3lim  1  3  n 
  3 1  
   n 
n 
 2
   
1
Here, as n    0
n
  1  0  2  0   
 3 1  3    31  0
  2  
  1 2   
 3 1  3    31
  2  
 31  3  3
 3[7]
 I1  21
4

Finding I2   xdx
1

4 1 3
Here, f (x)  x ; a  1 ; b  4 ; h  
n n

Class XII Maths www.vedantu.com 104


4
1  3   2.3   (n  1).3  
 I2   xdx  (4  1)lim f (1)  f 1    f 1    ...  f 1  
1
n 
n  n  n   n 
1   3   2.3   (n  1).3  
 3lim 1  1    1    ...  1  
n 
n  n  n   n 
1 3 3 3 
 3lim (1  1  1  ...  1) n times    2.  ...  (n  1).  
n 
n n n n 
1 3 
 3lim  n  (1  2  3  ...  (n  1)) 
n 
n n
It is known that the sum of n-terms is –
n(n  1)
1  2  3  ...  (n  1) 
1 3  n(n  1)   2
 3lim  n   
n 
n n  2  
‘n’ is a common factor
1  3  (n  1)  
 3lim .n 1   
n 
n  n  2  
 3  (n  1)  
 3lim 1   
n 
 n  2 
   n  1  
  3 n  
 3lim 1     
n 
  2 
   

   1  
  3 1  n   
 3lim 1     
n 
  2 
  

1
Here, as n     0
n
  31  0   
 3 1   
  2 
  3 
 3 1    
  2 
 23
 3 
 2 

Class XII Maths www.vedantu.com 105


5
 3 
2
15

2
15
 I2 
2
Now, I  I1  I2
15 42  15 27
 I  21   
2 2 2
4
27
Thus,  (x 2  x)dx 
1 2
1

5. Evaluate the definite integral as limit of sum –  exdx


1

Ans: The definite integral as a limit of sum is given by –


b
1
a f (x)dx  (b  a)lim
n 
f (a)  f (a  h)  ...  f (a  (n  1)h 
n
ba
With, h   0 as n  
n
1  (1) 1  1 2
Here, f (x)  e x ; a  1 ; b  1 ; h   
n n n
  2  2 
 f (  1)  f   1    f  1  2.   ...
1
1  n  n

  e dx  (1  (1))lim
x

1
n 
n   2 

  f  1  (n  1).  
  n 
1  1  1 n2   2
 1 2.   1 ( n 1).  
 2

 (1  1)lim
n  e  e e  n
 ...  e  n

n 
1  1 1  n2   2
1  n 
2.

1 
( n 1).  
2

 2lim
n  e  e e  e e  ...  e e n

n 
1  1  2
 
4
   (n 1).   
 2

 2lim e 1  e  e  ...  e
n n  n

n 
n  
2

It is known that the sum of n-terms in Geometric progression with a  1 ; r  e n


2n

e n 1 e2  1
is – 2  2

e 1
n
e 1
n

Class XII Maths www.vedantu.com 106


1  1  e 2  1 
 2lim e  2 
n 
n   e n  1 
  
1 e2  1 
 2e lim  2
1

n 
n  e n  1
 
 
 
 
 
1  e2  1 
 2e  
  2  
 lim  e  1  2 
n

 n   2  
  
 n  
eh  1
It is known that – lim 1
h 0
h
1  e  1 
2

 2e  
 (1)2 
 e 1 (e 2  1)
 (e1  e 1 )
1
e
e
1
1
 e dx  e  e
x
Thus,
1

6. Evaluate the definite integral as limit of sum –  (x  e 2x )dx


0

Ans: The definite integral as a limit of sum is given by –


b
1
a f (x)dx  (b  a)lim
n 
f (a)  f (a  h)  ...  f (a  (n  1)h 
n
ba
With, h   0 as n  
n
40 4
Here, f (x)  x  e2 x ; a  0 ; b  4 ; h  
n n

Class XII Maths www.vedantu.com 107


  4  4  4 
 f (0)  f  0    f  0  2.   f  0  3.   ... 
4
1  n  n  n

  (x  e )dx  (4  0)lim
2x
n 
n   4 
  
0
  f 0 (n 1)  
  n 
1  4 2. 4   4 2.2. 4   4 2(n 1) n4  
 4lim  0  e0     e n    2.  e n   ...   (n  1)  e 
n 
n n   n   n 
1  0 8
2.
8
(n 1) 
8
4 4 4 
 4lim  e  e  e  ...  e
n n n
    2.  ...  (n  1)  
n 
n   n n n 
1  8 8
(n 1) 
8
4 
 1  2  ...  (n  1)  
2.
 4lim 1  e  e  ...  e
n n n

n 
n   n 
n(n  1)
It is known that the sum of n-terms is – 1  2  3  ...  (n  1) 
2
8

Also, the sum of n-terms in Geometric progression with a  1 ; r  e n is –


8n

e n 1 e8  1
8  8

e 1
n
e 1
n

1  e8  1  4  n(n  1)  
 4lim  8   
n 
n  e n  1  n  2  
  
1   e8  1  
 4lim  8   2  n  1 
n 
n  e n  1  
 
1  e8  1  1
 4lim  8   4lim  2  n  1 
n 
n  en 1  n 
n
 
 
 
 
 
 e8  1   n  1
 4   4lim 2
  8   n 
 n 
 lim  e  1  8 
n

 n   8  
  
 n  

Class XII Maths www.vedantu.com 108


 
 
 
 
 e8  1   1
 4   4lim 2 1  
  8   n 
 n
 lim  e  1  8 
n

 n   8  
  
 n  

1
Here, as n    0
n
eh  1
And, lim 1
h 0
h
 e8  1 
 4   42 1  0
 (1)8 
 e8  1 
   4(2)
 2 
e8  1
 8
2
e8  1  16

2
 e8  15 
 
 2 
4
e8  15
Thus,  (x  e )dx 
2x

0 2

Exercise 7.9
1

1. Evaluate the definite integral–  (x  1)dx


1

Ans: The second fundamental theorem of integral calculus states that –


b

 f (x)dx  F(b)  F(a)


a

x2
Here,  (x  1)dx   x
2

Class XII Maths www.vedantu.com 109


1
1
 x2 
So,  (x  1)dx    x 
1 2  1
12
  (1) 2

   1    (1) 
2   2 
1  1 
   1    1
2  2 
1 1
 1 1 2
2 2
1
Thus,  (x  1)dx 2
1

3
1
2. Evaluate the definite integral–  x dx
2

Ans: The second fundamental theorem of integral calculus states that –


b

 f (x)dx  F(b)  F(a)


a

1
Here,  x dx  log x
3
1 3
So, 2 x dx  
 log x 
2
 log 3   log 2 
3
 log
2
3
1 3
Thus, 2 x dx  log
2
2

3. Evaluate the definite integral–  (4x 3  5x 2  6x  9)dx


1

Ans: The second fundamental theorem of integral calculus states that –


b

 f (x)dx  F(b)  F(a)


a

 x 4   x3   x 2 
Here,  (4x  5x  6x  9)dx  4    5    6    9x
3 2

 4  3  2
3
5x
 x4   3x 2  9x
3

Class XII Maths www.vedantu.com 110


2
2
 4 5x 3 
So,  (4x  5x  6x  9)dx   x 
3 2
 3x 2  9x 
1  3 1
 5(2) 3
  5(1) 3

  24   3(2)2  9(2)   14   3(1)2  9(1) 
 3   3 
 40   5 
 16   12  18  1   3  9
 3   3 
 40   5
  46    13  
 3  3
40 5
 46   13 
3 3
35
 33 
3
99  35

3
64

3
2
64
Thus,  (4x 3  5x 2  6x  9)dx 
1 3

4
4. Evaluate the definite integral–  sin 2xdx
0

Ans: The second fundamental theorem of integral calculus states that –


b

 f (x)dx  F(b)  F(a)


a

 cos 2x
Here,  sin 2xdx 
2
 
4
  cos 2x  4
So,  sin 2xdx  
0  2  0
   
  cos 2     cos 0
  4    
 
 2   2 
 

Class XII Maths www.vedantu.com 111


   
  cos  2    1 
   
 
 2  2
 
0 1
  
2 2
1

2

4
1
Thus,  sin 2xdx 
0 2


2
5. Evaluate the definite integral–  cos 2xdx
0

Ans: The second fundamental theorem of integral calculus states that –


b

 f (x)dx  F(b)  F(a)


a

sin 2x
Here,  cos 2xdx 
2
 
2
 sin 2x 
2
So,  cos 2xdx  
0  2  0
   
 sin 2  2    sin 0 
   
 
 2   2 
 
 sin    0 
 
 2   2 
00
0

2
Thus,  cos 2xdx  0
0

6. Evaluate the definite integral–  e xdx


4

Class XII Maths www.vedantu.com 112


Ans: The second fundamental theorem of integral calculus states that –
b

 f (x)dx  F(b)  F(a)


a

Here,  ex dx  ex
5
5
So,  ex dx  e x  4
4

 e5   e4 
 e 4 (e  1)
5
Thus,  e x dx  e4 (e  1)
4


4
7.  tan xdx
0

Ans: We know that,


 tan xdx   log cos x  C
Therefore, by second fundamental theorem of calculus

4 

 tan xdx  
0
 log cos x 
4
0


4
  
  tan xdx    log cos  log cos0 
0  4 

4
 1 
  tan xdx    log  log 1 
0  2 

4
1
  tan xdx  log 2
0 2


4
8.  cosecxdx

6

Ans: We know that,


 cosecxdx  log cosecx  cot x  C
Therefore, by second fundamental theorem of calculus

Class XII Maths www.vedantu.com 113



4 

 cos ecxdx  log cos ecx  cot x 



4

6
6

4
    
  cos ecxdx  log cos ec  cot  log cos ec  cot 
  4 4 6 6
6

4
  cos ecxdx  log 2  1  log 2  3 

 
6

4  2 1 
  cos ecxdx  log  

 2 3
6

1
1
9.  dx
1  x2
0

Ans: We know that,


1
 1  x 2 dx  sin x  C
1

Therefore, by second fundamental theorem of calculus


1
1 1
0 1  x 2   
1
dx  sin x 0
1
1
 dx  sin 1 1  sin 1 0 
0 1 x 2

1
1  
 dx   2  0 
0 1  x2
1
1 
 dx 
0 1  x2 2

1
1
10.  1 x
0
2
dx
Ans: We know that,
1
 1  x 2 dx  tan x  C
1

Therefore, by second fundamental theorem of calculus


1
1 1
0 1  x 2 dx   tan x  0
1

Class XII Maths www.vedantu.com 114


1
1
 dx   tan 1 1  tan 1 0
0 1 x
2

1
1  
 dx    0 
0 1 x 4 
2

1
1 
 dx 
0 1 x
2
4
3
1
11.
2
x1
dx
2

Ans: We know that,


1 1 x 1
 x 2  1 dx  2 log x  1  C
Therefore, by second fundamental theorem of calculus
3
3
1 1 x 1 
2 x 2  1 dx  2 log x  1 
 2
3
1 1 3 1 2 1 
 dx  log  log
2 x 1
2
2  3 1 2  1 
3
1 1 1 1
  2 dx  log  log 
2 x 1 2 2 3
3
1 1 3
  2 dx  log
2 x 1 2 2

4
12.  cos 2 xdx
0

Ans: We know that,


 1  cos 2x 
 cos xdx    2  dx
2

x sin 2x
  cos 2 xdx  
2 4
Therefore, by second fundamental theorem of calculus
 
4
 x sin 2x  4
0   2  4 
2
cos xdx
0

4
1   sin  sin 0 
  cos 2 xdx    0
0 2 2 2 2 

Class XII Maths www.vedantu.com 115



4
1  
  cos 2 xdx    0  0  0 
0 2 2 

4

  cos 2 xdx 
0 4

3
xdx
13.
2
x
2
1
Ans: We know that,
xdx 1  2x 
 x 2  1  2   x 2  1  dx
 log 1  x 2 
xdx 1
 2
x 1 2
Therefore, by second fundamental theorem of calculus
3

   
xdx 3

2 x 2  1    log 1  3 2
 log 1  2 2

2
3
xdx 1
 2  log10  log5
2 x 1 2
3
xdx 1 10
 2  log
2 x 1 2 5
3
xdx 1
 2  log 2
2 x  1 4
1
2x  3
14. 0 5x2  1 dx
2x  3
Ans: Solving  5x
2
1
dx ,

2x  3 1 5  2x  3
 5x 2  1 dx  5  5x 2  1 dx
2x  3 1 10x  15
 5x 2  1 dx  5  5x 2  1 dx
2x  3 1 10x 1
 5x 2  1 dx  5  5x 2  1 dx  3 5x 2  1 dx
2x  3 1 10x 1
 5x 2  1 dx  5  5x 2  1 dx  3  2 1  dx
5 x  
 5

Class XII Maths www.vedantu.com 116


2x  3
 5x 2
1
dx  log  5x 2  1 
1
5
3
 
5
tan 1 5 x

Therefore, by second fundamental theorem of calculus


2x  3 1
   1 1 
1
3 3
0 5x 2  1 dx   5 log  5  1  5 tan 5 x    5 log 1  5 tan 0
1

   
1
2x  3 1 3
 2 dx  log 6  tan 1 5
0 5x  1 5 5
1

15.  xex dx
2

Ans: Let x 2  t ,
Differentiating it we get,
2xdx  dt
Therefore, the integral becomes,
11 t
20
 e dt
1
11 t 1 t 
20
 e dt   2 e 
0
1
1 t 1 1
20
 e dt  e  e0
2 2
1
 e  1
2
2
5x 2
16.  2 dx
1 x  4x  3

Ans: The given integral can be written as


2
5x 2 2
 20x  15 
1 x 2  4x  3 dx  1 5  x 2  4x  3  dx
2
5x 2 2
20x  15
1 x 2  4x  3 dx  5x 1  1 x 2  4x  3 dx …(1)
2

2
20x  15
Solving  2 dx ,
1 x  4x  3

Let 20x  15  A  x 2  4x  3  B
d
dx
Equating the coefficients of x and constant term we get,
A  10,B  25

Class XII Maths www.vedantu.com 117


Let x 2  4x  3  t
Differentiating it we get,
 2x  4  dx  dt
Therefore, the integral becomes
dt dx
10  25
 x  2   12
2
t
dt dx 1  x  2  1 
10  25  10log t  25  log  
 x  2   12  x  2  1 
2
t 2
2
20x  15
2
 1  x  1  
 2 dx  10log  x 2  4x  3  25  log   
1 x  4x  3  2  x  3   1
2
20x  15 1 3 1 2
 dx  10log15  10log8  25  log  log 
1 x  4x  3 2 4
2
5 2
2
20x  15
 2 dx  10log5  10log3  10log 4  10log 2
1 x  4x  3

25
  log3  log5  log 2  log 4
2
2
20x  15 45 45 5 5
 2 dx  log5  log 4  log3  log 2
1 x  4x  3 2 2 2 2
2
20x  15 45 5 5 3
 2 dx  log  log
1 x  4x  3 2 4 2 2
Substituting it in (1) we get,
2
5x 2  45 5 5 3
1 x 2  4x  3 dx  5   2 log 4  2 log 2 
2
5x 2 5 5 3
 2 dx  5  9log  log 
1 x  4x  3 2 4 2

  2sec x  x 3  2  dx
4
2
17.
0

Ans: We know that,


x4
  2sec x  x  2  dx  2 tan x  4  2x
2 3

Therefore, by second fundamental theorem of calculus


 4

  2sec x  x 3  2  dx   2 tan x   2x 
2 x 4

 4 0

Class XII Maths www.vedantu.com 118


  1 
2
 
   2sec x  x  2  dx  2 tan     2     2 tan 0  0  0 
2 3

 4 4 4  4 
  4

   2sec2 x  x 3  2  dx  2 tan  5 
4 4 2
 4
   2sec2 x  x 3  2  dx  2  
2 1024



x x
18.   sin 2  cos 2  dx
0 2 2
Ans: We know that,

 2x 2 x 

 2x 2 x 
0  2
sin  cos
2
 dx   
0
 cos  sin  dx
2 2

 x x 

    cos 2  sin 2  dx    cos xdx


0 2 2 0

 cos xdx  sin x  C


Therefore, by second fundamental theorem of calculus

 cos xdx  sin   sin0


0

  cos xdx  0
0

2
6x  3
19.
0
x2
4
dx
Ans: Solving the integral we get,
6x  3 2x  1
 x 2  4 dx  3 x 2  4 dx
6x  3 2x 1
 x 2  4 dx  3 x 2  4 dx  3 x 2  4 dx
6x  3
dx  3log  x 2  4   tan 1
3 x
 2
x 4 2 2
Therefore, by second fundamental theorem of calculus
6x  3
2
2
 3 1 x 
0 x 2  4 dx  3log  x  4   2 tan 2 
2

0
2
6x  3  0
dx  3log  22  4   tan 1  3log  02  4   tan 1 
3 2 3
 2
0 x  4  2 2 2 2

Class XII Maths www.vedantu.com 119


6x  3
2
3 3
 dx  3log8  tan 1 1  3log 4  tan 1 0
0 x 4
2
2 2
2
6x  3 3 
 2 dx  3log8     3log 4  0
0 x  4 2 4 
2
6x  3 8 3
 2 dx  3log 
0 x 4 4 8
2
6x  3 3
 2 dx  3log 2 
0 x 4 8

1
 x 
20.   xe x  sin  dx
0 4 
Ans: Solving the integral we get,
 x 
x   d  x    cos 
1
 x
0  xe  sin 4  dx  x  e dx    dx x   e dx  dx    4 
x

   
 4 
1
 x x  4 x
0  xe  sin 4  dx  xe   e dx   cos 4
x x

1
 x  4 x
   xe x  sin  dx  xe x  e x  cos
0 4   4
Therefore, by second fundamental theorem of calculus
1
 x x   1 1 4   0 0 4 
0  xe  sin 4  dx  1e  e   cos 4    0e  e   cos0 
1
 x   4 2 2
   xe x  sin  dx  1   
0 4     

3
1
21.  1 x
1
2
dx


A.
3
2
B.
3

C.
6

D.
12
Class XII Maths www.vedantu.com 120
Ans: Solving the integral we get,
3
1
1 1  x 2 dx  tan x
1

Therefore, by second fundamental theorem of calculus


3
1
1 1  x 2 dx  tan 3  tan 1
1 1

3
1  
 dx  
1 1 x
2
3 4
3
1 
 dx 
1 1 x
2
12
Thus, the correct option is (D)
2
3
1
22.  4  9x
0
2
dx


A.
6

B.
12

C.
24

D.
4
Ans: Solving the integral we get,
1 1
 4  9x 2 dx   22  3x 2 dx
 
Let 3x  t ,
Differentiating it we get,
3dx  dt
1 1  3x 
 2 2 dx  tan 1  
2   3x  6  2 
Therefore, by second fundamental theorem of calculus
2
3
1 1 1  3 2  1 1
 4  9x
0
2
dx 
6
tan     tan 0
2 3 6
2
3
1 1 
 4  9x
0
2
dx  
6 4

Class XII Maths www.vedantu.com 121


2
3
1 
 dx 
0 4  9x
2
24
Thus, the correct answer is (C).

Exercise 7.10

Solve the following integrals.


1
x
1.  2 dx
0 x 1

Ans: Let x 2  1
Differentiating 2xdx  dt ,
Therefore, the integral becomes
1
x 1 2 dt
0 x 2  1 dx  2 1 t
1
x 1 2
 2 dx  log t 1
0 x 1 2
1
x 1
 2 dx  log 2  log1
0 x 1 2
1
x 1
 2 dx  log 2
0 x 1 2

2
2. 
0
sin  cos 5 d

Ans: The integral can be written as:


 
2 2


0
sin  cos 5 d   sin  cos 4  cos d
0

2
64
  sin  cos 5 d 
0 231
Let sin   t
Differentiating it we get,
cos d  dt
Therefore, the integral becomes

1

sin  cos d   t 1  t 2  dt


2
2

5

0 0

Class XII Maths www.vedantu.com 122



1

  sin  cos d   t 1  t 4  2t 2  dt


2
5

0 0

2 1
 1 9 5

  sin  cos5 d    t 2  t 2  2t 2  dt
0 0 
1
  3 11 7 
2
 t 2
t 2
2t 2 
 sin  cos d   
5

3 11 7 
0
 
2 2 2 0

2
2 2 4
  sin  cos 5 d   
0 3 11 7

2
154  42  132
  sin  cos5 d 
0 231

2
64
  sin  cos 5 d 
0 231

 2x 
1

 sin
1
3.  2 
dx
0 1 x 
Ans: Let x  tan 
Differentiating it we get,
dx  sec2 d
Therefore, the integral becomes

1
1  2x   2 tan   2
4

0  0 2  sec d
1
sin  2 
dx sin  
1 x   1   tan   

1
 2x  4

 sin
1
 2 
dx   sin 1  sin 2  sec 2 d
0 1 x  0

1  2x 
1 4

0 sin  1  x 2  dx  20  sec d


2

Let u  
And v  sec2 
Using integration by parts we get,

1
1  2x    d   4
0 sin  1  x 2  dx  2  sec d    d    sec d d
2 2

   0
Class XII Maths www.vedantu.com 123

1
 2x 
  sin  1
2 
dx  2  tan    tan d  04
0 1 x 
1  2x 
1 

  sin  2 
dx  2 
 tan   log cos   04
0 1 x 
1
 2x     
  sin 1  2 
dx  2  tan  log cos  log cos0 
0 1 x  4 4 4 
1
 2x   1 
  sin 1  2 
dx  2   log  log1
0 1 x  4 2 
1
 2x  
  sin 1   dx   log 2
1 x 
2
0 2

4. x
0
x  2dx

(Put x  2  t 2 )
Ans: Let x  2  t 2
Differentiating it, dx  2tdt
Therefore, the integral becomes
2 2

x x  2dx   2t 2  t 2  2  dt
0 2
2
2
 t 5 2t 3 
  x x  2dx  2  
0 5 3  2
2
 35 16 4 2 4 2 
  x x  2dx  2     
0  5 3 5 3 
2

  x x  2dx 
16 2 2  1  
0 15


2
sin x
5.  1  cos
0 x
dx 2

Ans: Let cos x  t


Differentiating it,  sin xdx  dt
Therefore, the integral becomes

0
2
sin x dt
0 1  cos 2 x dx   1 1  t 2

Class XII Maths www.vedantu.com 124



2
sin x 0
 1  cos
0
2
x
dx    tan 1 t 1

2
sin x
 dx    tan 1 0  tan 1 1
0 1  cos x
2


2
sin x 
 dx 
0 1  cos x
2
4

2
1
6.  x4x
0
2
dx
Ans: The integral can be written as
2 2
1 dx
0 x  4  x 2 dx  0 
1  17 
2

  x    
 2 4
2 2
1 dx
0 x  4  x 2 dx  0 
1 
2
 17   2

   x  
  2   2 
 
1
Let x   t
2
Differentiating it we get,
dx  dt
Therefore, the integral becomes
3
2
1 2
dt
x4x 2
dx   2

1  17 
   
0
 
2
2 t
 2  
 
3

  17  
2

3    t
2  
 log 
2
dt 1
 
1  
17 
2
   17   17  
2 
   t 
2
 2    t
 2     2   2   1
  2

Class XII Maths www.vedantu.com 125


  17  3  17  1 
3
     
1   2  2 2  2
 log 
2
dt
  log
1  
2
   17  3  17  1 
17  17

2 
  t      
2

 2     2  2  2  2 
 
3
2
dt 1  17  3 17  1 
  2

  log
17  3
 log 
17  1 
1  17  17 
    
2
2
 t
 2  
 
3
2
dt 1  17  3 17  1 
  2

 log
17  3
 
17  1 
1  17  17 
   t 
2
2
 2  
 
3
2
dt 1  25  17  10 17 
  2

  log 
1  17  17  8 
    
2
2
 t
 2  
 
2
1 1  21  5 17 
 dx  log 
0 x 4x
2
17  4 
1
1
7.
1
2 x
 2x  5
dx
Ans: The integral can be written as
1 1
1 1
 2
1 x  2x  5
dx  
1  x  1  2
2 2
dx

Let x  1  t
Differentiating it we get,
dx  dt
Therefore, the integral becomes
1 2
1 dt
1 x 2  2x  5 dx  0 t 2  22
2
1
1 1 1 t 
 2 dx   tan 
1 x  2x  5 2 2 0
1
1 1 1 
 2 dx   tan 1 1  tan 1 0 
1 x  2x  5 2 2 

Class XII Maths www.vedantu.com 126


1
1 
 dx 
1 x  2x  5
2
8

2
1 1  2x
8.   x  2x
1
2 

e dx

Ans: Let 2x  t
Differentiating it 2dx  dt ,
Therefore, the integral becomes
2
1 1  2x 1 1 2 2  t
1  x 2x 2 
 e dx      e dt
2 2  t t2 
2
1 1  2x 1
1 1 
    2  e dx     2  e t dt
1 x 2x  2 t t 
1
Let  f  t 
t
1
Then, f '  t    2
t
1
1 1  t 1

2  t  t 2  e dt  2 f  t   f '  t  e dt


t

1
1 1 
    2  e t dt  e t f  t   2
4

2 t t 
4
1
1 1  t  et 
    2  e dt   
2 t t   t 2
1
1 1  t e4 e2
    2  e dt  
2 t t  4 2
2
1 1  2x e2  e2  2 
    2  e dx 
1 x 2x  4

x  x 
1
1 3 3

9. The value of the integral 1 x4


dx
3

A. 6
B. 0
C. 3
D. 4
Ans: Let x  sin 
Differentiating it, dx  cos d

Class XII Maths www.vedantu.com 127


Therefore, the integral becomes

x  x   sin   sin  
1 1
1 3 3 3 3
2


1 x 4
dx   1 sin 4 
cos d
sin 1
3 3
 
 sin   sin  
1 1 2
2
3 3 2
 sin    cos  
3 3
  1 sin 4 
cos d   1 sin 4 
cos d
sin 1 sin 1
3 3
 
 sin   sin  
1 5
2
3 3 2
 cos   3
  sin  4
cos d   5 cosec 2d
sin 1
1
3
sin 1
1
3
sin 
3

 
 sin   sin  
1
3 3
2 2 5
  1 sin  4
cos d    cot  
1
3 cosec 2d
sin 1 sin 1
3 3

Let cot   t
Differentiating it,  cosec2d  dt
Therefore, the integral becomes
 x  x3 3
1
1 0 5


1 x4
dx   t dt
3

2 2
3

 x  x3 3
1
1
3 83
 dx  8
1 x4 8
3

 x  x3 3
1
1
3
 4
dx   16
1 x 8
3

 x  x3 3
1
1

 dx  6
1 x4
3

Thus, the correct answer is A.


x

10. If f  x    t sintdt ,then f '  x  is


0

A. cosx  xsinx
B. xsinx
C. xcos x
D. sinx  xcosx

Class XII Maths www.vedantu.com 128


x
Ans: Given f  x    t sin tdt
0

Using Integration by parts, we get


x x
 d  
f  x   t  sin tdt    t   sin tdt  dt
0 0  dt  
x

 f  x    t   cos t  0    cot tdt


x

 f  x     t  cos t   sin t  0
x

 f  x    x cos x  sin x
Therefore,
f '  x      x sin x  cos x   cos x
 f '  x   x sin x
Thus, the correct answer is B.

Exercise 7.11

Solve the following integrals.



2

 cos
2
1. xdx
0

2
Ans: Given I   cos 2 xdx …(1)
0

We know that,
a a

 f  x  dx   f  a  x  dx
0 0

Therefore, the integral becomes



2
 
I   cos 2   x  dx
0 2 

2
 I   sin 2 xdx …(2)
0

Adding equation (1) and (2),


2I    sin 2 x  cos 2 x  dx
2

Class XII Maths www.vedantu.com 129



2
 2I  1dx
0

 2I   x 02

 2I 
2

I
4

2

  cos 2 xdx 
0 4


2
sin x
2. 
0 sin x  cos x
dx

2
sin x
Ans: Given I   dx …(1)
0 sin x  cos x
We know that,
a a

 f  x  dx   f  a  x  dx
0 0

Therefore, the integral becomes


 
  x

sin
2
2 
I dx
   
sin   x   cos   x 
0

2  2 

cos x
2
I dx …(2)
0 sin x  cos x
Adding equation (1) and (2),

2
sin x  cos x
 2I   dx
0 sin x  cos x

2
 2I  1dx
0

 2I   x 02

Class XII Maths www.vedantu.com 130



 2I 
2

I
4

2
sin x 
 dx 
0 sin x  cos x 4

 3
2
sin 2 xdx
3.  3 3
0
sin x  cos x
2 2

 3
2
sin xdx
2
Ans: Given I   3 3 …(1)
0
sin x  cos x
2 2

We know that,
a a

 f  x  dx   f  a  x  dx
0 0

Therefore, the integral becomes


2 
3
 
  x  dx

sin
2 
2
I 3
  3
 
0
sin 2   x   cos 2   x 
2  2 
 3
2
cos xdx 2
I 3 3 …(2)
sin x  cos x
0 2 2

Adding equation (1) and (2),


 3 3
2
sin x  cos x
2 2
 2I   3 3 dx
0
sin x  cos x
2 2


2
 2I  1dx
0

 2I   x 02

 2I 
2

I
4

Class XII Maths www.vedantu.com 131


 3
2
sin xdx 2

 3 3 
0
sin x  cos x
2 2 4

cos 5 xdx
2
4.  5
0 sin x  cos x
5

cos5 xdx 2
Ans: Given I   5 …(1)
0 sin x  cos x
5

We know that,
a a

 f  x  dx   f  a  x  dx
0 0

Therefore, the integral becomes


 

cos5   x  dx
2 
2
I
   
0
sin 5   x   cos5   x 
2  2 

sin 5 xdx
2
I 5 …(2)
0 sin x  cos x
5

Adding equation (1) and (2),


sin 5 x  cos 5 x
2
 2I   5 dx
0 sin x  cos x
5


2
 2I  1dx
0

 2I   x  2
0


 2I 
2

I
4

2
cos 5 xdx 
 5 
0 sin x  cos x
5
4

5.  x  2dx
5

Class XII Maths www.vedantu.com 132


5
Ans: Let I   x  2dx
5

Since,  x  2   0 for interval  5, 2 .


Therefore,  x  2   0 for interval  2,5 .
b c b
As,  f  x dx   f  x dx   f  x  dx
a a c
2 5
Hence,    x  2 dx    x  2  dx
5 2
.
Thus,
2 5

I     x  2 dx    x  2  dx
5 2
2 5
x 2
  x2 
    2x     2x 
2  5  2  2
  2 2
 5 
2
   5 2  2 
2

   2  2   2  5      2  5   2  2  
 2 2   2 2 
 25   25 
  2  4   10     10  2  4 
 2  2 
25 25
 2  4   10   10  2  4
2 2
 29
8

6.  x  5dx
2
8
Ans: Let I   x  5dx
2

Since,  x  5   0 for interval  2,5 .


Therefore,  x  5   0 for interval 5,8 .
b c b

As,  f  x dx   f  x dx   f  x  dx


a a c
5 8

Hence,    x  5dx    x  5 dx .
2 5

Thus,
5 8

I     x  5dx    x  5  dx
2 5

Class XII Maths www.vedantu.com 133


5 8
 x2   x2 
    5x     5x 
2 2  2 5
  5 2
 2
2
   8 2  5
2

   5  5   5  2     5 8   5  5 
 2 2   2 2 
 25   25 
    25  2  10   32  40   25
2   2 
25 25
   25  2  10  32  40   25
2 2
9
1

 x 1  x  dx
n
7.
0
1
Ans: Let I   x 1  x  dx
n

Thus, I   1  x  1  1  x   dx
1
n

0
a a
Since,  f  x  dx   f  a  x  dx .
1 0

Therefore,
1

 1  x  x  dx
n

0
1

   x n  x n 1  dx
0
1
 x n 1 x n2 
  
 n  1 n  2 0
 1 1 
 
 n  1 n  2 

 n  2    n  1
 n  1 n  2 
1

 n  1 n  2 

4
8.  log  1  tan x  dx
0

Class XII Maths www.vedantu.com 134



4
Ans: Let I   log 1  tan x  dx
0
a a
Since,  f  x  dx   f  a  x  dx
0 0

 4
 
Therefore, I   log 1  tan   x   dx
0  4 
   
4
 tan  tan x 
  log 1  4
  dx
0
 1  tan tan x 
 4 

4
 1  tan x 
  log 1   dx
0  1  tan x 

4
2
  log dx
0 1  tan x
 
4 4
  log 2dx   log 1  tan x dx
0 0

4
  log 2dx  I
0

2I   x log 204

2I  log 2
4

I  log 2
8
2

9. x
0
2  xdx
2

Ans: Let I   x 2  xdx


0
a a

Since,  f  x dx   f  a  x dx


0 0
2

Therefore, I    2  x  xdx
0

Class XII Maths www.vedantu.com 135


2

  2x1 2  x 3 2 dx
0
2
  x3 2  x5 2 
 2   
  3 2  5 2 0
2
4 2 
  x3 2  x5 2 
3 5 0
4 32 2 52
  2   2
3 5
8 2 8 2
 
3 5
40 2  24 2

15
16 2

15

2
10.   2log sin x  log sin 2x  dx
0

2
Ans: Let I    2log sin x  log sin 2x  dx
0

 I    2logsin x  log  2sin x cos x   dx


2

0

2
 I    2log sin x  log sin x  log cos x  log 2  dx
0

2
 I    logsin x  log cos x  log 2  dx …….(1)
0

Since,
a a

 f  x  dx   f  a  x  dx
0 0

Therefore,

2
 I    log cos x  logsin x  log 2  dx …….(2)
0

On adding equation 1 and 2-

Class XII Maths www.vedantu.com 136



2
2I     log 2  log 2  dx
0

2
 2I  2log 2  dx
0


 I   log 2  
2

 I    log 2
2

 I    log 2
2
  1
 I  log 
2  2
 1
 I  log
2 2

2

 sin
2
11. xdx

2

2
Ans: Let I   sin 2 xdx

2

Since, sin 2 x is an even function.


 
2 2

 sin xdx  2  sin 2 xdx


2
Therefore ,
 0
2
a a
As if f  x  is an even function, then  f  x  dx  2 f  x  dx .
a 0

Hence,

2
I  2  sin 2 xdx
0

2
1  cos 2x
 2 dx
0 2

Class XII Maths www.vedantu.com 137



2
  1  cos 2x  dx
0

 sin2x  2
 x 
 2  0


2

x
12.  1  sin x dx
0

x
Ans: Let I   dx …….(1)
0 1  sin x

a a
Since,  f  x  dx   f  a  x  dx
0 0

Therefore, I  

   x  dx
0 1  sin x    x 

I

   x  dx …..(2)
0 1  sin x
On adding equation 1 and 2-


2I   dx
0 1  sin x

 2I   

1  sin x  dx
0 1  sin x   1  sin x 
 2I  

1  sin x  dx
0 cos 2 x

 2I   sec2 x  tan xsec x dx


0

 2I    2
I

2

 sin
7
13. xdx .

2

Class XII Maths www.vedantu.com 138



2
Ans: Let I   sin 7 xdx

2

Since, sin 7 x is an even function.



2

 sin xdx  0
2
Therefore ,

2
a
As if f  x  is an odd function, then  f  x  dx  0 .
a

Hence, I  0
2

 cos
5
14. xdx .
0

Ans: Let
2

I   cos5 xdx
0

cos  2  x   cos5 x …..(1)


5

2a a
If f  2a  x   f  x  then  f  x  dx  2 f  x  dx .
0 0
2a
If f  2a  x   f  x  then  f  x  dx  0
0

Since, cos    x    cos x


5 5

Therefore,
2

I  2  cos5 xdx
0

 I  2  0
I0

2
sin x  cos x
15.  1  sin xcos x dx
0

2
sin x  cos x
Ans: Let I   dx ……(1)
0 1  sin x cos x

Since,
a a

 f  x  dx   f  a  x  dx
0 0

Therefore,

Class XII Maths www.vedantu.com 139


   

sin   x   cos   x 
2  2  dx
2
I
   
0
1  sin   x  cos   x 
2  2 

cos x  sin x
2
I dx ……(2)
0 1  cos x sin x

On adding equation 1 and 2



2
0
 2I   dx
0 1  cos x sin x

I0

16.  log  1  cos x  dx


0

Ans: Let

I   log 1  cos x  dx …….(1)


0

Since,
a a

 f  x  dx   f  a  x  dx
0 0

Therefore,

 I   log 1  cos    x   dx
0

 I   log 1  cos x  dx …….(2)


0

On adding equation 1 and 2-


2I   log 1  cos x   log 1  cos x dx


0

 2I   log 1  cos 2 x  dx
0

 2I   logsin 2 xdx
0

 2I  2 logsin xdx
0

 I   logsin xdx …..(3)


0

Since, sin    x   sin x

Class XII Maths www.vedantu.com 140



2
Therefore, I   log sin xdx ……(4)
0


2

 I  2  logsin   x  dx
0 2 

2
 I  2  log cos xdx …….(5)
0

On adding equation 4 and 5-



2
2I    log sin x  log cos x  dx
0

2
 I    logsin x  log cos x  log 2  log 2  dx
0

2
 I    log 2sin x cos x  log 2  dx
0
 
2 2
 I    log 2sin x cos x  dx   log 2dx
0 0

Let 2x  t
On differentiating-
2dx  dt
If x  0 then t  0 .
Thus,
I 
 I   log 2
2 2
I 
   log 2
2 2
 I   log 2

a
x
17. 
0 x  ax
dx
a
x
Ans: Let I   dx …….(1)
0 x  ax
Since,
a a

 f  x  dx   f  a  x  dx
0 0

Class XII Maths www.vedantu.com 141


Therefore,
a
ax
I dx …….(2)
0 ax  x
On adding equation 1 and 2-
a
x  ax
2I   dx
0 ax  x
a

 2I   dx
0

 2I   x 0
a

 2I  a
a
I
2
4

18.  x  1 dx
0
4
Ans: Let I   x  1 dx
0

Thus,  x  1  0 when 0  x  1 and  x  1  0 when 1  x  4


b c b
Since,  f  x dx   f  x  dx   f  x 
a a c
1 4
Therefore, I   x  1dx   x  1 dx
0 1
1 4

 I     x  1dx    x  1 dx
0 1
1 4
 x  x 2
 2

 x      x 
 2 0  2 1
1  4
2
1
1   4  1
2 2 2
1 1
1  8  4  1
2 2
5
a a

19. Show that  f  x  g  x dx  2 f  x dx , if f and g are defined as


0 0

f  x   f  a  x  and g  x   g  a  x   4 .

Class XII Maths www.vedantu.com 142


a
Ans: Let  f  x g  x  dx …….(1)
0

Since,
a a

 f  x  dx   f  a  x  dx
0 0

Therefore,
a

  f  a  x g  a  x  dx
0
a

  f  x g  a  x  dx ……(2)
0

On adding equation 1 and 2-


2I   f  x  g  x   f  x  g  a  x dx
a

 2I   f  x g  x   g  a  x dx
a

As, g  x   g  a  x   4 .
Thus,
a

 2I   4f  x  dx
0
a

 I  2 f  x  dx
0
a a
Hence,  f  x  g  x dx  2 f  x dx , if f and g are defined as f  x   f  a  x  and
0 0

g  x   g a  x   4 .

 x  xcos x  tan 5 x  1 dx is
2
3
20. The value of

2

a. 0
b. 2
c. 
d. 1

 x  x cos x  tan 5 x  1 dx
2
Ans: Let I  3


2

Class XII Maths www.vedantu.com 143


   

  x  dx    x cos x  dx    tan x  dx   1 dx


2 2 2 2
I 3 5

   
2 2 2 2
a a
If f  x  is an even function, then  f  x  dx  2 f  x dx
a 0
a
And f  x  is an odd function, then  f  x  dx  0
a

Thus,

2
I  0  0  0  2  dx
0

 2 x 2
0


 2 
2

Hence, the correct option is C.

 4  3sin x 
2
21. The value of    dx is
0  4  3cos x 

a. 2
3
b.
4
c. 0
d. 2

 4  3sin x 
2
Ans: Let I     dx …..(1)
0  4  3cos x 

Since,
a a

 f  x  dx   f  a  x  dx
0 0

Therefore,
  

2
4  3sin   x 
 I    2   dx
0  
 4  3cos   x
 2 

Class XII Maths www.vedantu.com 144


 4  3cos x 
2
 I    dx …….(2)
0  4  3sin x 

On adding equation 1 and 2-



2
2I   log1dx
0

2
 2I   0dx
0

I0
Hence, the correct option is C.

Miscellaneous Exercise

1
1.
x  x3
1
Ans: Given
x  x3
1 1
So, 
xx 3
x 1  x 2 
1

x 1  x 1  x 
1 A B C
Let    …….(1)
x 1  x 1  x  x 1  x  1  x
 1  A 1  x 2   Bx 1  x   Cx 1  x 
 1  A  Ax 2  Bx  Bx 2  Cx  Cx 2
On equating the coefficients of x 2 , x and constant term –
A  B  C  0
BC 0
A 1
1 1
Thus, we get A  1,B  and C  
2 2
By equation 1-
1 1 1 1
  
x 1  x 1  x  x 2 1  x  2 1  x 
1 1 1 1 1 1
 dx   dx   dx   dx
x 1  x 1  x  x 2 1  x  2 1  x 

Class XII Maths www.vedantu.com 145


1 1
 log x  log 1  x   log 1  x 
2 2
1 1
1 1
 log x  log 1  x   log 1  x  2
2

2 2
 
 x C
 log
 1  x 12 1  x 12 
   
1

 x 2 2

 log  C
 1  x 2  
 
1  x 2

 log  2 
C
2 1 x 

1
2.
xa  xb
1
Ans: Given
xa  xb
1 xa  xb
 
xa  xb xa  xb
xa  xb

 x  a    x  b
xa  xb

ab

xa  xb
1
dx 
1
ab
 
 x  a  x  b dx
 3 3 

1   x  a   x  b2 
2
  
ab 3 3 

 2 2 
2  x  a 32  x  b 32   C
    
3 a  b   

1  a
3.  Hint: x= t 
x ax  x 2  
Ans: Given

Class XII Maths www.vedantu.com 146


1
x ax  x 2
a
Let x 
t
On differentiating-
a
dx   2 dt
t
1 1  a 
 dx   2 
 2 dt 
x ax  x 2 a a a  t 
a.   
t t t
1 1
  . dt
at 1 1

t t2
1 1
  2 dt
a t t2

t t2
1 1
  dt
a t 1
1
   2 t  1  C
a
Substituting value of t-
1 a 
  2  1  C
a x 
2 ax
  C
a x 
2 ax
  C
a x 

1
4. Integrate the expression: .
x  x +1
3
2 4 4

Ans:
1
The given expression is, .
x  x  1
3
2 4 4

Class XII Maths www.vedantu.com 147


On multiplying and dividing by x 3 , the following can be obtained as shown
below,
x  1 x  1
-3 -3 -3 -3
1  x4  1  4 1 
-3 4 4
x -3
x 4 4
1 4
   5  4   5 1  4 
x  1 x x  x  x 
3 -3
2
x .x -3 4 4 x 2 - x -3 5 4 4 x  x 
1
Now, consider t
x4
4 dt
 
x 5 dx
dx dt
 5 
x 4
-3
1 1 1  1 3

 
4
 dx   5 
1  4 
dx   1  t 4 dt

x 2  x 4  1 4 x  x 
3
4

 1 

1    C   1  1  4  C ,
1
1  1  t 4 
 dx     4 
where C is any
x 2  x 4  1 4  x 
3
4  1 
 4 
arbitrary constant.

1 1 1
5. Integrate the expression: . [Hint: = Put, x=t 6
 
1 1 1 1 1 1
x +x
2 3
x +x
2 3
x  1+x 
3 6

 
1
Ans: The given expression is, 1 1 .
x x 2 3

Observe the given hint and obtain as shown below,


1 1
 1 1 
3 
1 1
x x
2 3
x 1  x 6 
 
Consider x  t 6

 x  t 6  dx  6t 5
1 1 6t 5 t3
 1 dx   1 dx   t 2 1  t  dt  6  1  t  dt

1 1
x2  x3 x 3 1  x 6 
 
Now on dividing, we can obtain as shown below,

Class XII Maths www.vedantu.com 148


 1 
dx  6   t 2  t  1 
1
 1 1
 1  t
 dt

x x
2 3

 t 3   t 2  
 6        t  log 1  t  dt
 3   2  
1 1 1
 1

 2x 2  3x 3  6x 6  6log 1  x 6   C
 
1 1
 1

 2 x  3x 3  6x 6  6log 1  x 6   C , where C is any arbitrary constant.
 

5x
6. Integrate the expression: .
(x+1)  x 2 +9 
5x
Ans: The given expression is, .
(x  1)  x 2  9 
Now consider it as shown below,
5x A Bx  C
   ...1
(x  1)  x 2  9   x  1  x 2  9 
 5x  A  x 2  9   Bx  C  x  1
 5x  Ax 2  9A  Bx 2  Bx  Cx  C
On equating the coefficients of x 2 , x and constant term, it can be obtained that,
A  B  0 ... 2 
B  C  5 ... 3
9A  C  0 ... 4 
And on solving these equations, the values of A,B,C can be obtained as,
1 1 9
A  ,B  ,C  respectively.
2 2 2
Now, from equation 1 it can be clearly obtained that,
5x  1 x  9 
 (x  1)  x 2  9  dx   2  x  1 2  x 2  9   dx
 
 
1 1 x 9 1
 log x  1   2 dx   2 dx
2 2 x 9 2 x 9
1 1  9  1  x
 log x  1  log x 2  9     tan 1  
2 4  2  3  3

Class XII Maths www.vedantu.com 149


1 1 3 x
 log x  1  log x 2  9  tan 1    C , where C is any arbitrary
2 4 2 3
constant.

sinx
7. Integrate the expression, that is, .
sin  x-α 
sin x
Ans: The given expression is, .
sin  x   
Now, substitute x    t
dx  dt
sin x sin  t    sin t cos   cos tsin
 dx   dt   dt
sin  x    sin t sin t
  cos   cot tsindt
sin x
 dx  t cos   sin  log sin t  C1   x    cos 
sin  x   
 sin  log sin  x     C1
sin x
 dx  x cos   sin  log sin  x      cos   C1
sin  x   
 x cos   sin  log sin  x     C
where C1 ,C are any arbitrary constants and C  C1   cos  .

e5logx -e4logx
8. Integrate the expression, that is, 3logx 2logx .
e -e
5log x 4log x
e -e
Ans: The given expression is, 3log x 2log x .
e -e
e5log x - e4log x e  e  1
4log x log x

 3log x 2log x  2log x log x  e2log x  x 2


e -e e  e  1
Now, integrate the given expression as shown below
e5log x - e4log x x3
  3log x 2log x dx   x dx   C , where C is any arbitrary constant.
2

e -e 3

cosx
9. Integrate the expression, that is, .
4-sin 2 x

Class XII Maths www.vedantu.com 150


cos x
Ans: The given expression is, .
4 - sin 2 x
Now, substitute sin x  t
cos xdx  dt
cos x dt 1  t  1  sin x 
 dx    sin    C  sin    C , where C is
4 - sin 2 x  2   t   2   2 
2 2

any arbitrary constant.

sin8 x-cos8 x
10. Integrate the expression, that is, .
1-2sin 2 xcos 2 x
sin 8 x - cos8 x
Ans: The given expression is, .
1- 2sin 2 x cos 2 x


sin 8 x - cos8 x

 sin 4 x - cos 4 x  sin 4 x  cos 4 x 
1- 2sin 2 x cos 2 x sin 2 x  cos 2 x - sin 2 x cos 2 x  sin 2 x cos 2 x


 sin 2 x  cos 2 x  sin 2 x - cos 2 x  sin 4 x  cos 4 x 
sin 2 x 1  cos 2 x   cos 2 x 1  sin 2 x 
   sin 2 x  cos 2 x  sin 4 x  cos 4 x 
   cos 2x
sin 4 x  cos 4 x
Now, integrate the given expression as shown below
sin 8 x - cos8 x  sin 2x
 2 2
dx    cos 2xdx   C , where C is any arbitrary
1- 2sin x cos x 2
constant.

1
11. Integrate the expression, that is, .
cos  x+a  cos  x+b 
1
Ans: The given expression is, .
cos  x  a  cos  x  b 
On multiplying and dividing by sin      , the following can be obtained as
shown below,
1  sin  a  b  
 
sin  a  b   cos  x  a  cos  x  b  
1  sin  x  a    x  b   
  
sin  a  b   cos  x  a  cos  x  b  

Class XII Maths www.vedantu.com 151


1  sin  x  a  cos  x  b   cos  x  a  sin  x  b  
  
sin  a  b   cos  x  a  cos  x  b  
1  sin(x  a) sin  x  b   1
     tan(x  a)  tan(x  b) 
sin  a  b   cos(x  a) cos(x  b)  sin(a  b)
1 1

cos  x  a  cos  x  b 
dx 
sin(a  b)
  tan(x  a)  tan(x  b)  dx
1 1  cos(x  b) 
 dx   log   C , where C is any
cos  x  a  cos  x  b  sin(a  b)  cos(x  a) 
arbitrary constant.

x3
12. Integrate the expression, that is, .
1-x 8
x3
Ans: The given expression is, .
1- x 8
Now, substitute x 4  t
 4x 3dx  dt
x3
sin  x   C , where C is any
1 dt 1 -1 1 -1 4
 dx    sin t  C 
1- x 8 4 1-  t 2 4 4
arbitrary constant.

ex
13. Integrate the expression, that is, .
 1+e  2+e 
x x

ex
Ans: The given expression is, .
1  e  2  e 
x x

Now, substitute e x  t
 e x dx  dt
ex dt  1 1 
 dx       dt
1  e  2  e 
x x
 t  1 t  2    t  1  t  2  
ex ex  1
 dx  log t  1  log t  2  C  log x  C , where C is
1  ex  2  ex  e 2
any arbitrary constant.

Class XII Maths www.vedantu.com 152


1
14. Integrate the expression, .
(x +1)  x 2 +4 
2

1
Ans: The given expression is, .
(x  1)  x 2  4 
2

Now consider it as shown below,


1 Ax  B Cx  D
 2   ...1
(x  1)  x 2  4   x 2  1  x 2  4 
 1   Ax  B  x 2  4    Bx  C   x 2  9 
 1  Ax 3  4Ax  Bx 2  4B  Cx 3  Cx  Dx 2  D
On equating the coefficients of x 3 , x 2 , x and constant term, it can be obtained that,
A  C  0 ... 2 
B  D  0 ... 3
4A  C  0 ... 4 
4B  D  1 ... 5 
And on solving these equations, the values of A,B,C,D can be obtained as,
1 1
A  0,B  ,C  0,D  respectively.
3 3
Now, from equation 1 it can be clearly obtained that,
1 1  1 1 
 (x 2  1)  x 2  4  dx     dx
3   x 2  1  x 2  4  
1 1 x
 tan 1 x  tan 1  C
3  3 2  2
1 1 x
 tan 1 x  tan 1  C , where C is any arbitrary constant.
3 6 2

15. Integrate the expression, that is, cos 3 xelogsinx .


Ans: The given expression is, cos3 xelogsin x .
Observe as shown below,
 cos3 xelogsin x  cos3 x sin x
Now, consider cosx  t
 sin xdx  dt
t 4  cos 4 x
  cos xe
3 logsin x
dx   cos x sin xdx    t dt 
3 3
C  C , where C is
4 4
any arbitrary constant.

Class XII Maths www.vedantu.com 153


16. Integrate the expression, that is, e3logx  x4 +1 .
-1

Ans: The given expression is, e3log x  x 4  1 .


-1

Observe as shown below,


x3
 e  x  1  4
3log x 4 -1

 x  1
Now, consider x 4  1  t
 4x 3dx  dt
x3 log x 4  1
  e  x  1 dx   4
-1 1 dt 1
3log x 4
dx    log t  C   C , where C
x 1 4 t 4 4
is any arbitrary constant.

17. Integrate the expression, that is, f'  ax+b   f(ax+b) .


n

Ans: The given expression is, f '  ax  b   f (ax  b)  .


n

Now, consider  f (ax  b)   t


 af '(ax  b)dx  dt
f (ax  b)
n 1
1 t n 1
  f '  ax  b  f (ax  b)  dx   t n dt  C  C , where
n

a a  n  1 a  n  1
C is any arbitrary constant.

1
18. Integrate the expression, that is, .
sin 3 xsin  x+α 
1
Ans: The given expression is, .
sin 3 x sin  x   
1 1
 
sin 3 x sin  x    sin 3 x sin x cos   cos  sin x
1

sin 4 x cos   sin 3 x sin  cos x
1 cosec 2 x
 
sin 2 x cos   sin  cot x cos   sin  cot x
Now, substitute cos   sin  cot x  t
 cosec2 x sin dx  dt

Class XII Maths www.vedantu.com 154


1 cosec 2 x 1 dt 2 t
 dx      C
sin 3 x sin  x    cos   sin  cot x sin  t sin 
1 2 cos   sin  cot x
 dx  C ,
sin x sin  x   
3
sin 

2 sin  cos x
 cos   C
sin  sin x
where C is any arbitrary constant.

sin-1 x-cos-1 x
19. Integrate the expression, that is, ,x   0,1 .
sin-1 x +cos-1 x
sin -1 x - cos-1 x
Ans: The given expression is, , x   0,1 .
sin -1 x  cos-1 x
sin -1 x - cos-1 x
Assume, I   dx
sin -1 x  cos-1 x

It is clearly known to us that, sin -1 x  cos-1 x 
2

 2cos 1 x 2  2 
 4
I   2 dx     2cos 1 x  dx  .  dx   cos 1 xdx
  2   2 
2
4
 I  x   cos 1 xdx ...1

Assume, I1   cos 1 xdx
Also, substitute x t
dx  2tdt

 1 t 2  1  t 2  2  t2 
 I1   cos xdx  2 cos t.   
1
2 
dt   t cos t   
1
2 
dt
 2  1  t  2   1  t 
 1  t 
 I1  t 2 cos 1 t   1  t 2 dt    2 
dt  t 2 cos 1 t  1  t 2  sin 1 t
 1 t  2 2

Now, from equation 1 it can be clearly obtained that,

Class XII Maths www.vedantu.com 155


4 2 t  1 
Ix t cos 1
t  1  t 2
 sin t 
  2 2 
4 x  1 
x  x cos 1
x  1  x  sin x 
 2 2 
4   1  
 I  x   x   sin 1 x   x  x 2  sin 1 x 
 2  2 2 
2

 I   x   2x  1 sin 1 x  
   2
x  x2  C

2  2x  1 1 2
I sin x  x  x 2  C , where C is any arbitrary constant.
 

1- x
20. Integrate the expression, that is, .
1+ x
1- x
Ans: The given expression is, .
1 x
1- x
Assume, I   dx
1 x
Now, substitute x  cos 2 
dx  2sin  cos dt

1- cos  2sin 2
I    2sin  cos   d    2 sin 2d

1  cos  2cos 2
2

   tan 2sin  cos d
2

sin
 I  2 2 2sin  cos  cos d  4 sin 2  (2cos 2   1)d
 2 2
 2 2
cos
2
    
 I  8 sin 2 cos 2 d  4 sin 2 d  2 sin 2 d  4 sin 2 d
2 2 2 2 2
 1  cos 2   1  cos     sin 2    sin  
 I  2   d  4    d  2     4  C
 2   2  2 2  2 2 
sin 2
 I     2sin   C    1  cos 2  cos   2 1  cos 2   C
2
Class XII Maths www.vedantu.com 156
 I  cos 1 x  x(1  x)  2 1  x  C  2 1  x  cos 1 x  x  x 2  C ,
where C is any arbitrary constant.

2+sin2x x
21. Integrate the expression, that is, e .
1+cos2x
2  sin 2x x
Ans: The given expression is, e .
1  cos 2x
2  sin 2x x
Assume, I   e dx
1  cos 2x
2  2sin x cos x x  1  sin x cos x  x
I e dx     e dx    sec x  tan x  e dx
2 x

 
2 2
2cos x cos x
Now, consider f  x   tan x
 f '  x   sec 2 xdx
2  sin 2x x
I   e dx    f  x   f '  x   e x dx  e x f  x   C  e x tan x  C , where
1  cos 2x
C is any arbitrary constant.

x 2 +x+1
22. Integrate the expression: .
(x+1)2  x+2 
x2  x  1
Ans: The given expression is, .
(x  1) 2  x  2 
Now consider it as shown below,
x2  x  1 A B C
    ...1
(x  1)  x  2   x  1  x  1  x  2 
2

 x 2  x  1  A  x  1 x  2   B  x  2   C  x  1
2

 x 2  x  1  A  x 2  3x  2   B  x  2   C  x 2  2x  1
 x 2  x  1   A  C  x 2  x  3A  B  2C    2A  2B  C 
On equating the coefficients of x 2 , x and constant term, it can be obtained that,
A  C  1 ... 2 
3A  B  2C  1 ... 3
2A  2B  C  1 ... 4 
And on solving these equations, the values of A,B,C can be obtained as,
A  2,B  1,C  3 respectively.
Now, from equation 1 it can be clearly obtained that,

Class XII Maths www.vedantu.com 157


x2  x  1 
 2 1 3  
 (x  1)2  x  2  dx    x  1  x  1  x  2   dx
  

 

1 1 1
 2 dx   2 dx  3 dx
x 1  x  1 x2
1
 2log x  1  3log x  2   C , where C is any arbitrary constant.
(x  1)

1-x
23. Integrate the expression, that is, tan -1 .
1+x
1- x
Ans: The given expression is, tan -1 .
1 x
1- x
Assume, I   tan -1 dx
1 x
Now, consider x  cos 
dx   sin d

1- cos  2sin 2
 I   tan -1   sin   d    tan 1 2 sin d

1  cos  2cos 2
2

   tan 1 tan sin d
2
1 1 1
 I     sin d      cos    1.( cos )d      cos    sin  
2 2 2
x
2
1
2
1
 
 I  cos 1 x  1  x 2  C  x cos 1 x  1  x 2  C , where C is any
2
arbitrary constant.

x 2 +1  log  x 2 +1 -2logx 


24. Integrate the expression, that is, .
x4
x 2  1 log  x 2  1 - 2log x 
Ans: The given expression is, .
x4

Class XII Maths www.vedantu.com 158


x 2  1 log  x 2  1 - 2log x 
Assume, I   dx
x4
x2  1 x 2  1   x 2  1 
I
x4 
 log  x 2
 1 - log x 2

  log
x 4   x 2  

1 x2  1   1 
 3  log 1  2  
x x   x 
2

1
Consider 1  2  t
x
2
 3 dx  dt
x
Now, integrate the given expression as shown below
x 2  1 log  x 2  1 - 2log x  1 x2  1   1 
I   dx   3  log 1  2   dx
x 4
x x   x 
2

1 1
  t 2 log tdt  C
2
Using integration by parts, it can be obtained that,
 3 3 
1  1
  d  1
   1  t 2
1 t 2

I  log t. t 2 dt   log t   t 2 dt  dt   log t.   . dt 
2   dt    2  3 t 3 
 2 2 
1  2 3 2 1  1 3 2 3 1 3  2
 I   t 2 log t   t 2 dt   t 2 log t  t 2  t 2 log t  
2 3 3  3 9 3  3
1  2 3
2 1
 1 3
2 3
1  3
2
 I   t 2 log t   t 2 dt   t 2 log t  t 2  t 2 log t  
2 3 3  3 9 3  3
1  1   1  2
 I  1  2   log 1  2     C , where C is any arbitrary constant.
2  x   x  3 

π  1-sinx 
25. Find the value of the expression, that is,  π
2
ex   dx .
 1-cosx 
  1  sin x 
Ans: The given expression is, 
2
ex   dx .
 1  cos x 
  1  sin x 
Assume, I   e x   dx
2  1  cos x 

Class XII Maths www.vedantu.com 159


 x x  2 x 
 1  2sin cos cosec

 I   e x  2 2  dx   e x  2  cot x  dx
x   
2 2

2
 2sin 2  2

 2   
x
Now, substitute f  x    cot
2
 1 x 1 x
 f '  x      cosec 2  dx  cosec 2 dx
 2 2 2 2

 x
 I   e  f (x)  f '(x)  dx  e f (x)    e x cot 
 x x 

2 2  2 
2

     
 

 I  e  cot  e 2 cot   0  e 2   e 2
 2 4  
π
sinxcosx
26. Find the value of the expression, that is,  0
4

sin 4 x+cos4 x
dx .

sin x cos x
Ans: The given expression is,  0
4

sin 4 x  cos 4 x
dx .

sin x cos x
Assume, I   4
dx
sin 4 x  cos 4 x
0

sin x cos x
 
cos 4
x tan x sec 2 x
 I  04
dx  0 4
dx
sin 4 x  cos 4 x 1  tan 4 x
cos 4 x
Now, substitute tan 2 x  t
 2 tan x sec2 xdx  dt

And also when x  0, t  0 and when x  , t  1 .
4
1 1 dt 1 1 1  
  tan 1 1  tan 1  0      
1
 I  0  
 tan 1
t 

2 1 t 2
2 0
2 2 4  8

π
cos 2 x
27. Find the value of the expression, that is,  0
2

cos 2 x+4sin 2 x
dx .

cos 2 x
Ans: The given expression is,  0
2

cos 2 x  4sin 2 x
dx .

cos2 x
Assume, I   2
dx
0
cos 2 x  4sin 2 x

Class XII Maths www.vedantu.com 160



cos 2 x 1 2 4  4  3cos 2 x
I 2
dx  0 dx
0
cos 2 x  4 1  cos 2 x  3 3cos 2 x  4
1 2 4  3cos 2 x 1 2 4 1 2 1 2 4sec2 x
 I   0 dx  0 dx   0 dx  0 dx
3 4  3cos 2 x 3 4  3cos 2 x 3 3 4sec 2 x  3
1 2 1 2 4sec 2 x  2 2 2sec 2 x
 I   x 0  0 dx    dx ...1
3 3 4 1  tan 2 x   3 6 3 0 1  4 tan 2 x 

2sec 2 x
Observe, 
0
2

1  4 tan 2 x 
dx

Now, substitute 2tan x  t


 2sec2 xdx  dt

And also when x  0, t  0 and when x  , t   .
2

2sec x2
dt 
    tan 1     tan 1  0   
 
 02 dx   dx  
 tan 1
t 

1  4 tan x 
2 0
1  t 
2 0
2
Henceforth from equation 1 , it can be obtained that,
 2   2 
I     
6 3 2  6 6 6

π
sinx+cosx
28. Find the value of the expression, that is,  π
6
2

sin2x
dx .

sin x  cos x
Ans: The given expression is,  
6
2

sin 2x
dx .

sin x  cos x
Assume, I  2 dx
6 sin 2x

sin x  cos x 
sin x  cos x
I 2
dx   2
dx
  1  1  2sin x cos x  1   sin 2 x  cos 2 x  2sin x cos x 
0 0


sin x  cos x
 I  02 dx
1   sin x  cos x 
2

Now, substitute  sin x  cos x   t


 (sin x  cos x)dx  dt
 1 3    3 1
And also when x  , t    and when x  , t   .
6  2  3  2 

Class XII Maths www.vedantu.com 161


3 1 3 1
dt dt
 I  1 23    21 3
2 1  t2 


2  1  t2
1 1 1
As  , it can be thus obtained that is an even function,
1   t  1  t2 1  t2
2

  a f  x  dx  2 0 f  x  dx
a a

3 1
dt 3 1  3 1  1 3 
 I  20 2
  2sin 1 t  0 2  2sin 1   x  , t    and
1  t2  2  6  2 
  3 1
when x  , t   .
3  2 

dx

1
29. Find the value of the expression, that is, 0
.
1+x- x
dx

1
Ans: The given expression is, .
0
1 x - x
dx
Assume, I  0
1

1 x - x
1 1 x  x 1 1 x  x
 I  0 dx  0
1
 dx
1 x - x 1 x  x 1 x  x
2 1 1
2  2  32  2  23  2 4 2
 I  0 1  xdx  0 xdx  1  x     x     2   1  
1 1
3

3 0 3  0 3   3 3

π
sinx+cosx
30. Find the value of the expression, that is, 0
4

9+16sin2x
dx .

sin x  cos x
Ans: The given expression is, 
0
4

9  16sin 2x
dx .
sin x  cos x

Assume, I   4
dx
9  16sin 2x
0

Now, substitute sin x  cos x  t


  cos x  sin x  dx  dt

And also when x  0, t  1 and when x  , t  0 .
4
 sin x  cos x   t
2 2

 1  2sin x cos x  t 2
 1  sin 2x  t 2

Class XII Maths www.vedantu.com 162


 sin 2x  1  t 2
dt dt dt
 I  1  1  1 2
0 0 0

9  16 1  t  25  16t  5   4t 
2 2 2

0
1 1 5  4t  1  1 1
I  log   log 1  log   log 9
4  2  5 5  4t  1 40  9  40

π
31. Find the value of the expression, that is, 0
2
sin2xtan -1  sinx  dx .

Ans: The given expression is, 
0
2
sin 2x tan 1  sin x  dx .

Assume, I  02 sin 2x tan 1  sin x  dx
 
 I  02 sin 2x tan 1  sin x  dx  02 2sin x cos x tan 1  sin x  dx
Now, substitute sin x  t
cos xdx  dt

And also when x  0, t  0 and when x  , t  1 .
2
 I  20 t tan  t  dt
1 1

Observe,  t tan 1  t  dt
 d  tan 1 t   t2 1 t2
  t tan  t  dt  tan t  tdt   
1 1
 
tdt dt  tan 1
t.   1  t 2 . 2 dt
 dt  2
2
t tan t1
t 1 1
2 2
t tan t 1
1 t 2 tan 1 t 1 1
  dt   1dt   dt   t  tan 1 t
2 1 t 2
2 1 t 2
2 2 2
1
 t tan t 1
2 1
1  1    1
 0 t.tan 1 tdt  
1
 t  tan 1 t     1    
 2 2 2 0 2  4 4 4 2
Henceforth from equation 1 , it can be obtained that,
 1 
I  2     1
2 2 2

π xtanx
32. Find the value of the expression, that is, 0
secx+tanx
dx .
x tan x
dx ...1 .

Ans: The given expression is, 
0
sec x  tan x
 x tan x
Assume, I  0 dx
sec x  tan x
Class XII Maths www.vedantu.com 163
   x  tan    x  dx  f x dx  f a  x dx 
       

 I  0
a a

sec    x   tan    x  0 0

    x  tan  x     x  tan  x  dx ... 2


 
 
I dx  
(sec  x   tan  x )
0
sec  x   tan  x 
0

Now, on adding equations 1 and  2  , it can be obtained that,


sin x
 x tan x 
cos x  sin x
2I  0 dx  0 dx   0 dx
sec x  tan x 1

sin x 1  sin x
cos x cos x
 sin x  1  1 1  1  sin x
dx   x 0  0
  
 2I  0 dx  0 dx  0 dx
1  sin x 1  sin x cos 2 x
 2I    x 0  0  sec 2 x  tan x sec x  dx  2    tan x  sec x 0
  

 2I  2   0   1  0  1  2  2


    2
I
2


4
33. Find the value of the expression, that is,  x-1 + x-2 + x-3  dx .
1

Ans: The given expression is, 1  x  1  x  2  x  3  dx .


4


4
Assume, 1
 x  1  x  2  x  3  dx
 I  1 x  1 dx  1 x  2 dx  1 x  3 dx
4 4 4

 I  I1  I 2  I3 ...1
where, I1  1 x  1 dx,I 2  1 x  2 dx,I3   1 x  3 dx
4 4 4

Now, consider, I1  1 x  1 dx , where  x  1  0 1  x  4


4

4
 x2   1  9
 I1  1  x  1 dx    x   8  4   1  ... 2 
4

2 1  2  2
Again, consider, I 2  1 x  2 dx , where  x  2   0  2  x  4 and
4

 x  2   0 1  x  2 .
4 4
 x2   x2 
 I 2  1  2  x  dx  2  x  2  dx   2x      2x 
2 4

 2 1  2 2
 1 1 5
 I 2   4  2  2    8  8  2  4   2  ...  3
 2 2 2

Class XII Maths www.vedantu.com 164


Also, consider, I3  1 x  3 dx , where  x  3  0  3  x  4 and
4

 x  3  0 1  x  3 .
3 4
 x2   x2 
 I3  1  3  x  dx  3  x  3 dx  3      3x 
3 4

 2 1  2 3
 9 1  9  1 5
 I3  9   3    8  12   9   2   ... 4 
 2 2  2  2 2
Now, from equations 1 ,  2  ,  3 and  4  it can be obtained that,
9 5 5 19
I   
2 2 2 2
 sin x  1  1 1  1  sin x
dx   x 0  0
  
 2I  0 dx  0 dx  0 dx
1  sin x 1  sin x cos 2 x
 2I    x 0  0  sec 2 x  tan x sec x  dx  2    tan x  sec x 0
  

 2I  2   0   1  0  1  2  2


    2
I
2

dx 2 2

3
34. Prove the following equation: = +log .
x  x+1 3
1 2
3
dx 2 2
Ans: The given equation is, 1 2
3
  log .
x  x  1 3 3
dx
Assume, 1 2
3

x  x  1
Now consider it as shown below,
1 A B C
 2   2 ...1
x  x  1 x x  x  1
 1  Ax  x  1  B  x  1  C  x 2 
 1  Ax 2  Ax  Bx  B  Cx 2
On equating the coefficients of x 2 , x and constant term, it can be obtained that,
A  C  0 ... 2 
A  B  0 ... 3
B  1 ... 4 
And on solving these equations, the values of A,B,C can be obtained as,
A  1,B  1,C  1 respectively.

Class XII Maths www.vedantu.com 165


Now, from equation 1 it can be clearly obtained that,
dx 3
 1 1 1  
I  1 
3
     dx
x  x  1 1 
2
 x x 2
 x  1 

3
   x 1 1 
3
 1 4 1
 I    log x   log  x  1   log      log     log  2   1
 x  1   x  x 1 3 3
2 2 2 2
 I  log 4  log3  log 2   log 2  log3   log   
3 3 3 3
Henceforth, it can be clearly proved.


4
35. Prove the following equation: 0
xe xdx=1 .


4
Ans: The given equation is, 0
xe x dx  1 .
Assume, I  0 xe x dx
4

Using integration by parts, it can be obtained that,


 d  x   x 
4  x 1 x 1
I  x 0 e x dx  0  
4
e     0   0  e  e  1  1
 xe    e

 dx  

Henceforth, it can be clearly proved.


-1
36. Prove the following equation: 1
x17cos 4 xdx=0 .
1
Ans: The given equation is, 1
x17 cos 4 xdx  0 .
1
Assume, I  1 x17 cos 4 xdx
Now, consider f (x)  x17 cos 4 x
 f  x    x  cos 4  x   x17 cos 4 x  f  x 
17

 f  x  is an odd function and henceforth it is clearly known to us that when


 f  x  dx  0 .
a
f (x) is an odd function, then a
1
 I  1 x cos xdx  0
17 4

Henceforth, it can be clearly proved.


π
2
37. Prove the following equation:
3

sin 3 xdx= .
0
2


2
Ans: The given equation is, 02 sin 3 xdx  .
3

Assume, I  02 sin 3 xdx

Class XII Maths www.vedantu.com 166


   
 I   sin x sin xdx   1  cos x  sin xdx   sin xdx   cos 2 x sin xdx
0
2 2
0
2 2
0
2
0
2

 cos3 x  2

1 2
 I    cos x    2
  1  
 3 0
0
3 3
Henceforth, it can be clearly proved.
π
38. Prove the following equation:  0
4
2tan 3 xdx=1-log2 .

Ans: The given equation is, 
0
4
2 tan 3 xdx  1- log 2 .

Assume, 
0
4
2 tan 3 xdx
   
 I   2 tan x tan xdx   1  sec x  tan xdx   tan xdx   sec 2 x tan xdx
0
4 2
0
4 2
0
4
0
4

 
 tan 2 x  4 
  2
 I  2   2  log cos x  4  1  2 log cos
  log cos0   1  log 2  log1
 2 0
0
4 0

 I  1  log 2
Henceforth, it can be clearly proved.

π

1
39. Prove the following equation: sin -1xdx= -1 . 0
2

Ans: The given equation is, 0 sin 1 xdx   1 .
1

2
Assume, I  0 sin 1 xdx
1

 I  0 sin 1 x.1dx
1

Using integration by parts, it can be obtained that,


1 1 1 1 1  2x 
I  sin 1 x.x  0  0  0 2 0
1
xdx  
 x sin 1
x   dx
1 x 2
1 x 2

Now, substitute 1  x  t 2

  2x  dx  dt
And also when x  0, t  1 and when x  1, t  0 .
1 1 dt 1 0 
  x sin 1 x  0   2 t   sin 1 1  1   1
1 1
 I   x sin 1 x  0  0
2 t 2 1 2
Henceforth, it can be clearly proved.


1
40. Evaluate 0
e 2-3x as a limit of sum.

Class XII Maths www.vedantu.com 167


Ans:
Assume, I  0 e 2-3x
1

It is clearly known that


1 ba
a f (x)dx   b  a  lim   where, h 
b

n 

 f a  f (a  h)  ...  f (a  (n  1)h) 
 ,
a n

In this case, a  0,b  1,f  x   e 23x


1 0 1
h 
n n
1
 0 e23x dx  1  0  lim f  0   f (0  h)  ...  f (0  (n  1)h) 
1

n 
n
1

n 
1
n n 
1
n
 
 0 e23x dx  lim e 2  e 23h  ...  e 23 n 1h   lim e 2 1  e 3h  e 6h ...  e 3 n 1h 

  3
.n  
1  2  1  e n  1  2  1  e3   2 3 1 1 
3   e  e  1 lim
 0 e dx  lim  
1 2 3x
e  lim e   3 
n   1  e n   n  n   1  e n  
3
n  n 
n  e n  1
       
 3 
1  n  e  e  1 e 1  e 2 1  2 1 
2 3

 0 e dx  e  e  1 lim  3  
1 2 3x 2 3
  e  
n 
3  e n  1 3 3 3 e
 

dx
41. The expression, that is, e x
+e-x
is equal to

A. tan-1  ex  +C
B. tan-1  e-x  +C
C. log  ex -e-x  +C
D. log  ex +e-x  +C
dx
Ans: The given expression is,  ex  e-x .
dx
Assume, I  
e x  e-x
Now, consider e x  t
 e x dx  dt
dx 1
I   x  dt   tan 1 t dt  C , where C is any arbitrary constant.
e e -x
1 t 2

Hence, the correct answer is option (A).


Class XII Maths www.vedantu.com 168
cos2x
42. The expression, that is,  dx is
 sinx+cosx 
2

-1
A. +C
sinx+cosx
B. log sinx+cosx +C
C. log sinx-cosx +C
1
D. 2 +C
 sinx+cosx 
cos 2x
Ans: The given expression is,  sin x  cos x 2 dx .
 
cos 2x
Assume, I   2 dx
 sin x  cos x 
(sin x  cos x)(cos x  sin x) (cos x  sin x)
I dx   dx
 sin x  cos x 
2
 sin x  cos x 
Now, substitute  sin x  cos x   t
 (cos x  sin x)dx  dt
1
 I  I   dt  log t  C  log cos x  sin x  C , where C is any arbitrary
t
constant.
Hence, the correct answer is option (B).

43. If f  a+b-x  =f  x  , then  xf  x  dx is equal to


b

a+b b
A.
2 a
 f  b-x  dx
a+b b
B.
2 a
 f  b+x  dx
b-a b
C.  f  x  dx
2 a
a+b b
D.
2 a
 f  x  dx
Ans: Assume, I  a xf  x  dx ...1
b

 I  a  a  b  x  f  a  b  x  dx  a f  x  dx  a f  a  b  x  dx 
b b b

 
 I  a  a  b  x  f  x  dx   a  b  a f  x  dx  I [using 1 ]
b b

Class XII Maths www.vedantu.com 169


 2I   a  b  a f  x  dx
b

a  b
 f  x  dx
b
I a
2
Hence, the correct answer is option (D).

-1  2x-1 

1
44. The value of tan  2 
dx is equal to
0
 1+x-x 
A. 1
B. 0
C. -1
π
D.
4
 2x -1 
Ans: Assume, I  0 tan -1 
1
2 
dx
1 x - x 
 x - 1  x  
 I  0 tan -1   dx  0 tan  x   tan 1  x  dx ...1
1 1 -1 1

 1  x 1  x  
I0
Hence, the correct answer is option (D).

Class XII Maths www.vedantu.com 170


Access of Full Crash Course to Revise
screen Syllabus Course Entire Syllabus

Test Series and Chapter Wise Course to


Assignment help you master One chapter

Notes & Recordings Unlimited In-class


of every class Doubt Solving

Use VPROP & Get Extra 20% OFF

Subscribe Now
https://vdnt.in/VPROPDF

GRADES 6-12 | CBSE | ICSE | JEE | NEET


Download
Vedantu Learning APP
Free LIVE
MasterClasses

Unlimited Access
to Study Materials

Specially Designed
Tests

DOWNLOAD
Best of 2019 Winner THE APP

USER’S CHOICE
APP AWARD 2019

Category
Everyday Essentials

W W W.V E D A N T U . C O M
 Thank You
for downloading the PDF

Samajh Aayega toh


Maza Aayega!!

FREE MASTER CLASS SERIES


 For Grades 6-12th targeting JEE, NEET, CBSE, ICSE & many more Register for FREE
 Free 60 Minutes LIVE Interactive classes everyday
Limited Seats!
 Learn from India’s Best Master Teachers

W W W.V E D A N T U . C O M

You might also like